You are on page 1of 82
Pediatric “Hee Orthopaedic PN al secel 4 2010 Pediatric Orthopaedic Examination Answer Book + 7 Figure 1a Figure Ib Question 1 ‘A 5-year-old boy has had a limp for the past 4 weeks with intermittent pain at the foot. He remains normally active and has no history of trauma. He has no fevers, rashes, or swelling. Examination reveals tendemess at the mid-dorsum of the foot medially. Radiographs are seen in Figures 1a and 1b. Treatment should include which of the following? MRI of the foot with gadolinium Open biopsy of the lesion ‘Needle aspiration and culture, followed by antibiotic treatment Observation or an orthotic arch support Steroid injection of the lesion yRepe PREFERRED RESPONSE: 4 DISCUSSION: Osteochondrosis of the tarsal navicular is most commonly identified between the ages of 2and 9 years. The condition is benign and self limited in nature. In patients with severe pain, a period of casting may be warranted, but otherwise management usually consists of observation or a supportive orthotic. REFERENCES: DiGiovanni CW, Patel A, Calfee R, et al: Osteonecrosis in the foot. J Am Acad Orthop Surg 2007;15:208-217. Williams GA, Cowell HR: Kohler’s disease of the tarsal navicular. Clin Orthop Relat Res 1981;158:53- 58. 8+ American Academy of Orthopaedic Surgeons Figure 2 Question 2 ‘A3-year-old girl has had pain and swelling in her left thigh for the past 3 weeks. Her mother states she has had a temperature as high as 100.4 degrees F (38 degrees C) and a weight loss of 5 pounds. A CBC shows a WBC count of 11,000/mm3, an erythrocyte sedimentation rate of 13 mnv/h, and a C-reactive protein of 0.3. A radiograph is shown in Figure 2. What is the next step in management? Biopsy and culture of the lesion MRI of the left femur IV antibiotics for 6 weeks Incision and drainage of the left femur Repeat radiograph in 3 months wpope PREFERRED RESPONSE: 2 DISCUSSION: The history and laboratory studies indicate that this is not an infection. A lesion in this location and in this age group is likely a Ewing’s sarcoma. The presentation is usually a painful mass. About 20% of patients have a fever. The radiograph shows a typical mottled, permeative lesion with periosteal reaction. An MRI scan should be obtained to further evaluate the sofi-tissue mass. Staging of the lesion should take place before biopsy, which should be done by the surgeon who would be performing the next stage of surgical treatment, ideally an orthopaedic oncologist. REFERENCES: Gibbs CP Jr, Weber K, Scarborough MT: Malignant bone tumors. Instr Course Lect 2002;51:413-428. Meyer JS, Nadel HR, Marina N, et al: Imaging guidelines for children with Ewing sarcoma and ‘osteosarcoma: A report from the Children’s Oncology Group Bone Tumor Committee. Pediatr Blood Cancer 2008;51:163-170. 2010 Pediatric Orthopaedic Examination Answer Book * 9 Figure 3 Question 3 A 9-year-old girl has had bilateral knee and leg pain for the past 2 years. The family has noted increasing deformity in both lower extremities. She is less than the fifth percentile for height. Examination reveals bilateral femoral bowing, mild medial-lateral laxity of the knees, and the deformities shown in the radiograph seen in Figure 3. What is the most likely diagnosis? 1, Renal osteodystrophy 2. Diastrophic dysplasia 3. Metaphyseal dysplasia 4, Osteogenesis imperfecta 5. Fibrous dysplasia PREFERRED RESPONSE: | DISCUSSION: The widening, bowing, and cupping of the physes indicate some form of metabolic bone disease; therefore, the most likely diagnosis is renal osteodystrophy. ‘The age of onset makes X-linked hypophosphatemic rickets less likely. The ground glass lesions and widening of the medullary canal characteristic of fibrous dysplasia are not present. There are no fractures creating the deformities indicating osteogenesis imperfecta. There is an asymmetry of the deformities that makes diastrophic dysplasia less likely. REFERENCES: Goldberg MJ, Yassir W, Sadeghi-Nejad A: Clinical analysis of short stature. J Pediatr Orthop 2002;22:690-696. Parmar VS, Stanitski DF, Stanitski CL: Interpretation of radiographs in a pediatric limb deformity practice: Do radiologists contribute? J Pediatr Orthop 1999;19:732-734. 10 American Academy of Orthopaedic Surgeons Question 4 Patients with slipped capital femoral epiphysis are more likely to experience a delay in definitive diagnosis if they initially present to a physician reporting which of the following problems? Limp Hip pain Knee pain Proximal thigh pain Buttock pain veeee PREFERRED RESPONSE: 3 DISCUSSION: A delay in diagnosis of slipped capital femoral epiphysis (SCFE) can lead to significant worsening of the deformity or even progression from a stable to an unstable SCFE, Those patients that report knee pain as their primary complaint are most likely to experience significant delay. Other variables associated with this delay include Medicaid insurance and stable SCFE, REFERENCES: Kocher MS, Bishop JA, Weed B, et al: Delay in diagnosis of slipped capital femoral epiphysis. Pediatrics 2004;113:e322-€325. Rahme D, Comley A, Foster B, et al: Consequences of diagnostic delays in slipped capital femoral epiphysis. J Pediatr Orthop B 2006;15:93-97. Question 5 Physiologic bowing of the lower extremities should spontaneously correct by what age? 3 months. 6 months 12 months 36 months 72 months PREFERRED RESPONSE: 4 veer DISCUSSION: Physiologic bowing is common and benign, Bowing is typically symmetric, involves both the femur and tibia, and is usually most prominent in toddlers. It usually resolves by 2 years of age but there is great variability. By age 36 months, almost all children will correct spontaneously. In children with physiologic bowing, the screening examination is typically normal and a family history is absent; therefore, radiographs are not necessary. If the deformity has not resolved by age 2 years, an AP radiograph of the lower limbs should be obtained. This provides documentation of the severity of the bowing, permits measurement of the metaphyseal-diaphyseal angle and/or Langenskiold grade, and allows evaluation for conditions such as rickets or bony dysplasia, No treatment is indicated for physiologic bowing. REFERENCES: Abel MF (ed): Orthopaedic Knowledge Update: Pediatrics 3, Rosemont, IL, American ‘Academy of Orthopaedic Surgeons, 2006, p 7. Salenius P, Vankka E: The development of the tibiofemoral angle in children. J Bone Joint Surg Am 1975;57:259-261. 2010 Pediatric Orthopaedic Examination Answer Book + I Question 6 A 2-year-old child refused to walk 3 days prior to being seen because of pain in the left hip. The pain has gradually subsided and the child is now walking. He is afebrile and has full motion of the hips. Laboratory studies show a normal CBC with differential and C-reactive protein. An ultrasound shows a joint effusion in the right hip. What is the most likely diagnosis? Juvenile inflammatory arthritis Septic arthritis Osteomyelitis of the femur Leukemia Toxic synovitis weUne PREFERRED RESPONSE: 5 DISCUSSION: The most likely diagnosis is toxic synovitis, and the normal C-reactive protein supports that diagnosis, Juvenile inflammatory arthritis is extremely rare to present with hip involvement. The child most likely does not have a bacterial infection because he has improved rapidly without treatment, A normal CBC with differential precludes the diagnosis of leukemia. REFERENCES: Herring JA (ed): Tachdji Saunders, 2008, pp 2068-2070. Del Beccaro MA, Champoux AN, Bockers T, et al: Septic arthritis versus transient synovitis of the hip: The value of screening laboratory tests. Ann Emerg Med 1992;21:1418-1422. Kocher MS, Mandiga R, Zurakowski D, et al: Validation of a clinical prediction rule for the differentiation between septic arthritis and transient synovitis of the hip in children. J Bone Joint Surg Am 200486: 1629-1635. ’s Pediatric Orthopaedics, ed 4. Philadelphia, PA, WB 12 + American Academy of Orthopaedic Surgeons Figure 7a Figure 7b Question 7 A 14-year-old boy is lifting weights and feels a sudden pain in his back, associated with sciatica bilaterally, The sciatica persists for several weeks. The radiograph shown in Figure 7a is negative, and the CT scan shown in Figure 7b is available for evaluation. An MRI scan is read as a disk bulge. Management should consist of resection of the fragment through a microdiskectomy approach. epidural steroid injections until symptoms improve. laminectomy with surgical excision of the limbus fragment. activity restrictions until the symptoms improve. chiropractic manipulation. pReNE PREFERRED RESPONSE: 3 DISCUSSION: A limbus or apophyseal fracture caused by heavy lifting or twisting is commonly seen in older children and adolescents. Patients deseribe feeling a popping sensation and report radicular symptoms. Radiographs usually are not sufficient to diagnose the injury. MRI or CT should be used to determine the exact location of the fracture. Nonsurgical management is rarely successful. A wide laminectomy with surgical excision of the limbus fragment is recommended if neurologic symptoms are present. REFERENCE: Fischgrund JS (ed): Orthopedic Knowledge Update 9. Rosemont, IL, American Academy of Orthopaedic Surgeons, 2008, p 694. 2010 Pediatric Orthopaedic Examination Answer Book * 13, ¥ Figure 8 Question 8 A 16-year-old football player reports the acute onset of pain in his left foot, An AP radiograph is shown in Figure 8, What treatment is most likely to result in successful healing for this injury? Restricted activity Short leg walking cast Short leg cast with no weight bearing Percutaneous fixation with a screw Open reduction and internal fixation with bone grafting wpeno PREFERRED RESPONSE: 4 DISCUSSION: Fractures in this region of the fifth metatarsal have been shown to be prone to delayed union and nonunion and therefore are most reliably managed with internal fixation. Bone grafting is gen- erally not required. REFERENCES: Herrera-Soto JA, Scherb M, Duffy MF, et al: Fractures of the fifth metatarsal in children and adolescents. J Pediatr Orthop 2007;27:427-43 1 Fetzer GB, Wright RW: Metatarsal shaft fractures and fractures of the proximal fifth metatarsal. Clin Sports Med 2006;25:139-150. Question 9 ‘A 13-year-old girl with adolescent idiopathic scoliosis is otherwise healthy with a normal neurologic ‘examination and she began her menstrual cycle 3 months ago. Standing radiographs show a high left thoracic curve from T1-T6 that measures 29 degrees, a right thoracic curve from T7-L1 that measures 65 degrees, and a left lumbar curve from L1-L5 that measures 31 degrees, correcting to 12, 37, and 10 degrees, respectively, on bending films. Her Risser sign is 1. What is the most appropriate management? 1. Bracing 2. Posterior spinal fusion of only the right thoracic curve 3. Posterior spinal fusion from T2-L4 4. Vertebral body stapling to halt progression of the curve 3. Anterior and posterior spinal fusion PREFERRED RESPONSE: 2 14+ American Academy of Orthopaedic Surgeons DISCUSSION: The patient has typical adolescent idiopathic scoliosis with a right thoracic curve. This represents a Lenke-1B curve pattern; therefore, only treatment of the thoracic curve is required, The proximal thoracic and thoracolumbar curves are very flexible. The patient is Risser 1 and has just started her menstrual cycles; therefore, she is at significant risk for further curve progression. Bracing is not appropriate for a curve of this magnitude and will not halt the progression of this curve, nor will vertebral body stapling stop this curve. Vertebral body stapling is sometimes useful in small thoracic curves of {ess than 35 degrees and skeletally immature patients. Anterior and posterior spinal fusion is not required because the patient has no other risk factors, such as neurofibromatosis nor is she at risk for crankshaft, Anterior fusion is an option, but itis not listed. REFERENCES: Lenke LG, Betz RR, Harmes J, et al: Adolescent idiopathic scoliosis: A new classification to determine extent of spinal arthrodesis. J Bone Joint Surg Am 2001;83:1169-1181 ‘Lenke LG, Betz RR, Haher TR, et al: Multisurgeon assessment of surgical decision making in adolescent idiopathic scoliosis: Curve classification, operative approach, and fusion levels. Spine 2001;26:2347- 2353. Question 10 The risk of progression with congenital kyphosis is greatest with which of the following? Anterior unsegmented bar Block vertebra Posterior hemivertebra Anterolateral bar and contralateral quadrant vertebrae Butterfly vertebra DRYERS PREFERRED RESPONSE: 4 DISCUSSION: The risk of neurologic compromise associated with congenital kyphosis is normally secondary to risk of progression. The classic study of the natural history of congenital spinal deformity by McMaster and Singh confirms that an anterolateral bar with contralateral quadrant vertebrae has the greatest risk. REFERENCES: McMaster MJ, Singh H: Natural history of congenital kyphosis and kyphoscoliosis: A study of one hundred and twelve patients. J Bone Joint Surg Am 1999;81:1367-1383. Herring JA (ed): Tachdjian's Pediatric Orthopaedics, ed 4. Philadelphia, PA, WB Saunders, 2008, p 351. 2010 Pediatrie Orthopaedic Examination Answer Book + 15 Question 11 Achondroplasia is caused by an abnormality of which of the following? Parathyroid-related protein (PTHrP) Bone morphogenic protein 2 Transforming growth factor-B (TGF-B) 4. Fibroblast growth factor receptor 3 (FGFR3) 5. Insulin-like growth factor binding proteins eRe PREFERRED RESPONSE: 4 DISCUSSION: Achondroplasia results from mutation of the fibroblast growth factor receptor 3. Bone morphogenic proteins are regulators of growth differentiation and morphogenic embryology. Anomalies of this protein are seen in increasing defects in limbs distally. Parathyroid-related protein is seen in Jensen-type metaphyseal chondrodysplasia. Transforming growth factors and the morphogenic proteins affect the production of matrix. REFERENCES: Leet Al, Chomey GS: The physis, in Cramer KE, Scher! SA, Einhorn TA (eds); Orthopaedic Surgery Essentials: Pediatrics. Philadelphia, PA, Lippincott Williams & Wilkins, 2004, pp 327-332, van der Eerden BC, Karperian M, Wit JM: Systemic and local regulators of the growth plate, Endocr Rev 2003;24:782-801. a Figure 12 Question 12 A 15-year-old boy is seen for an injury to his left shoulder, Examination reveals that he has pain and deformity of the proximal humerus. The skin in the area is intact and there is no neurovascular ‘compromise involving the extremity. An AP radiograph is seen in Figure 12, What is the most appropriate treatment? Shoulder immobilizer Hanging arm cast Closed reduction and a shoulder spica cast |. Closed reduction and percutaneous pin fixation 5. Hemiarthroplasty PREFERRED RESPONSE: 4 Beye 16 + American Academy of Orthopaedic Surgeons DISCUSSION; In adolescents with proximal humeral physeal fractures and minimal remodeling potential, closed reduction and pin fixation has been shown to be safe and results in excellent long-term shoulder function. Satisfactory reduction is not likely to result from the use of a hanging arm cast or be maintained by a shoulder immobilizer or a spica cast following closed reduction. Hemiarthroplasty is not indicated for this injury. REFERENCES: Dobbs MB, Luhmann SL, Gordon JE, et al: Severely displaced proximal humeral epiphyseal fractures. J Pediatr Orthop 2003;23:208-215, Bahrs C, Zipplies S, Ochs BG, et al: Proximal humeral fractures in children and adolescents. J Pediatr Orthop 2009;29:238-242. Figure 132 Figure 13b Question 13 ‘A 12-year-old child falls from his bicycle and injures his right knee, Evaluation in the emergency department reveals knee effusion and pain with extremes of range of motion. Radiographs are shown in Figures 13a and 13b. Attempts at closed reduction are made and he is placed in a long leg cast with the knee flexed at 10 to 20 degrees. At follow-up, repeat radiographs continue to show anterior displacement of the fracture, What structure is most likely entrapped under the fragment? Anterior fat pad Anterior cruciate ligament Posterior cruciate ligament Anterior horn of the medial meniscus Anterior hom of the lateral meniscus yeep PREFERRED RESPONSE: 4 2010 Pediatric Orthopaedic Examination Answer Book * 17 DISCUSSION: Avulsion fractures of the tibial spine are a relatively rare injury in children. Historically, the most common cause of this fracture was falls from bicycles, but with the increased participation in competitive sports, the etiology is changing. Most fractures occur in children ages 8 to 14 years, and they typically’ present with a painful hemarthrosis and refusal to bear weight. The Meyers and McKeever classification is based on degree of displacement, where type I is minimally displaced, type II is anteriorly displaced with an intact posterior hinge, and type III is completely displaced. The Illa and IIIb modifications have been added to account for fragment comminution and rotation, respectively. Long leg casting is advocated for type I fractures, though there is debate whether the knee should be maintained in full extension or in 10 to 20 degrees of flexion. Management of type Il and III fractures is much more controversial. Type II fractures can be treated closed if adequate reduction can be achieved, but if not, surgical management is indicated. Surgery is also indicated for type III fractures, and results of open versus arthroscopic procedures are similar long term. Kocher and associates examined 80 consecutive skeletally immature patients with type II or III tibial eminence fractures that were treated surgically, They found that the anterior horn of the medial meniscus was entrapped beneath the displaced fracture fragment in 36 of 80 cases, whereas the lateral meniscus was only entrapped in 1 of 80 cases. This is not to be confused with the data from Lowe and associates in JBJS 2002 where they found the lateral meniscus to be involved in blocking reduction. ‘This was not thought due to entrapment of the lateral meniscus. Rather, with the anterior cruciate ligament and lateral meniscus still being attached to the avulsed fracture fragment, they felt the two structures were pulling in opposite disections and therefore blocking reduction of the fragment, REFERENCES: Falstie-Jensen S, Sondergard-Petersen PE: Incarceration of the meniscus in fractures of the intercondylar eminence of the tibia in children. Injury 1984;15:236-238. Kocher MS, Micheli LJ, Gerbino P, et al: Tibial eminence fractures in children: Prevalence of meniscal entrapment. Am J Sports Med 2003;31:404-407. Accousti WK, Willis RB: Tibial eminence fractures. Orthop Clin North Am 2003;34:365-375. Lowe J, Chaimsky G, Freedman A, et al: The anatomy of tibial eminence fractures: arthroscopic observations following failed closed reduction. J Bone Joint Surg Am 2002;84:1933-1938. 18+ American Academy of Orthopaedic Surgeons Figure 14a Figure 14b, Figure 14c Question 14 A 14-year-old boy underwent in situ screw fixation for a left slipped capital femoral epiphysis 8 months ago. He noted 3 months of intermittent right hip pain but is presently asymptomatic. The last episode of pain was 2 days prior to this office visit, He reports that he has pain approximately once a week over the past 3 months. Examination of the right hip is normal, and includes pain-free internal rotation. Radiographs and an MRI scan are shown in Figures 14a through 14c, Treatment should consist of which of the following? In situ screw fixation of the right hip Physical therapy Limitation of activities and return to the clinic if pain persists Biopsy of the femoral neck lesion Irrigation and debridement of the right hip yeere PREFERRED RESPONSE: 1 DISCUSSION: The patient history is concerning for a pre-slip slipped capital femoral epiphysis (SCFE) of the right hip. In one study, nearly 40% of patients with SCFE had bilateral involvement, and of that 40%, half presented initially with a unilateral SCFE but had a subsequent SCFE on the contralateral limb. Radiographs are normal, but the MRI scan shows increased signal about the proximal femoral physis. ‘Treatment should include prophylactic screw fixation of the right hip. REFERENCES: Aronsson DD, Loder RI, Breur GJ, et al: Slipped capital femoral epiphysis: Current concepts. J Am Acad Orthop Surg 2006;14:666-679. Loder RT, Aronson DD, Greenfield ML: The epidemiology of bilateral slipped capital femoral epiphysis: A study of children in Michigan. J Bone Joint Surg Am 1993;75:1141-1147. Loder RT: Controversies in slipped capital femoral epiphysis. Orthop Clin North Am 2006;37:211-221, vii 2010 Pediatric Orthopaedic Examination Answer Book + 19 Figure 15a Figure 15b Question 15 When first seen in the emergency department, the patient with the injury seen in Figures 15a and 15b was not able to extend the wrist or the thumb, What is the best initial management? Closed reduction and casting, with expected nerve injury recovery with time 2. Closed reduction and percutaneous pinning, with expected nerve injury recovery with time Immediate open reduction with internal fixation and exploration of the radial nerve Immediate open reduction with internal fixation and exploration of the median nerve Immediate open reduction with internal fixation and exploration of the ulnar nerve Aa PREFERRED RESPONSE: 2 DISCUSSION: The injury is a type 3 supracondylar humerus fraacture with a radial nerve injury. Most nerve injuries associated with fractures recover spontaneously within 6 to 12 weeks, Complete recovery is expected within 3 to 6 months. Closed reduction and percutaneous pinning is the recommended treatment for supracondylar fractures of the elbow. Cast treatment for displaced fractures carries a higher risk of associated compartment syndrome, There is no indication for exploring the radial nerve acutely. Open reduction is necessary only if the closed reduction fails. REFERENCES: Abel MF (ed): Orthopaedic Knowledge Update: Pediatrics 3. Rosemont, IL, American ‘Academy of Orthopaedic Surgeons, 2006, pp 406-409 Campbell CC, Water PM, Emans JB, et al: Neurovascular injury and displacement in type IT supracondylar humerus fractures. J Pediatr Orthop 1995;15:47-52. 20+ American Academy of Orthopaedie Surgeons Figure 16 Question 16 Ina fracture such as the one shown in Figure 16 (Salter-Harris type I fracture of the distal femur), which of the following best describes the location of the fracture? 1. The fracture occurs through the zone of hypertrophy of the physis. . The fracture occurs through the zone of proliferation of the physis. 3. The fracture is generally confined to the germinal zone, which explains the high rate of growth arrest in these fractures. 4. The fracture generally propagates through multiple layers of the physis. 5. The fracture is generally confined to the zone of endochondral ossification, PREFERRED RESPONSE: 4 DISCUSSION: The growth plate in the distal femur has an undulating topography, with prominences called mammillary bodies that interdigitate with other portions of the physis to provide stability at the dis- tal femur, A typical distal femoral physeal fracture propagates through multiple layers of the growth plate as opposed to most Salter-Harris type I physeal fractures. REFERENCES: Smith DG, Geist RW, Cooperman DR: Microscopic examination of a naturally occurring epiphyseal plate fracture. J Pediatr Orthop 1985;5:306-308, Jaramillo D, Kammen BF, Shapiro F: Cartilaginous path of physeal fracture separations: Evaluation with MR imaging: An experimental study with histologic correlation in rabbits. Radiology 2000;215:504-511 2010 Pediatric Orthopaedic Examination Answer Book + 21 Question 17 ‘A 14-year-old boy has had a 3-month history of low back pain with no known trauma. The pain is worse with activity and relieved by rest, although he does report difficulty with prolonged sitting in school. The patient was on the football team but stopped participating because of the back pain during football practice, He reports no history of radicular pain and denies any numbness, tingling, or weakness in the legs. Neurologic examination is normal, Back examination reveals slight tendemess over the lower back area but no swelling or skin defects. Strength testing is 5 over 5 in the lower extremities and the straight leg raise test is negative. Back range of motion is nearly full, but back extension is painful. The hamstrings are slightly tight. Initial radiographs, including AP, lateral and oblique views, are negative. ‘What is the best test to determine the patient's diagnosis? Flexion and extension lateral radiographs MRI Myelogram. Diskogram Bone scan with SPECT paeye PREFERRED RESPONSE: 5 DISCUSSION: A bone scan with SPECT is very sensitive and specific for spondylolysis not seen on initial radiographs. MRI can sometimes visualize spondylolysis, but it is not as sensitive nor as specific as a bone scan with SPECT. Flexion and extension views have no role in the evaluation of the patient who presents with classic spondylolysis-type symptoms. The most sensitive physical examination finding is, pain with back extension, Oblique radiographs can be obtained; but they are not as sensitive or specific as a bone scan with SPECT. The patient does not have any signs of a disk problem; therefore, an evaluation of the disk is not helpful. REFERENCES: Hu SS, Tribus CB, Diab M, et al: Spondylolisthesis and spondylolysis. J Bone Joint Surg Am 2008;90:656-671 Lawrence JP, Greene HS, Grauer JN: Back pain in athletes. J Am Acad Orthop Surg 2006;14:726-735. 22+ American Academy of Orthopaedic Surgeons Figure 18 Figure 18a Question 18 A 14-year-old boy is involved in a motor vehicle accident and sustains the injury shown in Figures 18a and 18b. What is the most likely diagnosis? Hawkins type I talar neck fracture Hawkins type HI talar neck fracture Hawkins type III talar neck fracture Hawkins type IV talar neck fracture Talar body fracture yeere PREFERRED RESPONSE: 2 DISCUSSION: Talar neck fractures are uncommon. In children younger than age 6 years, displacement is rare and closed treatment is usually successful in achieving union and avoiding osteonecrosis. In adolescence, however, talar neck fractures should be treated as they are in adults. This fracture is displaced, and there is dislocation of the subtalar joint. The tibiotalar and talonavicular joints remain reduced, In the classification originally created by Hawkins and modified by Canale and Kelly, this would bea Hawkins type II, carrying a 20% to 50% risk of osteonecrosis. ‘The rate of osteonecrosis increases with the Hawkins grade. The presence of talar neck comminution and open talar neck fractures are also risk factors for osteonecrosis after talar neck fracture, REFERENCES: Vallier HA, Nork SE, Barei DP, et al: Talar neck fractures: Results and outcomes. J Bone Joint Surg Am 2004;86:1616-1624. Jensen I, Wester JU, Rasmussen F, et al: Prognosis of fracture of the talus in children: 21 year follow up of 14 cases. Acta Orthop Scand 1994;65:398-400. Bucholz RW, Heckman JD, Court-Brown C (eds): Rockwood and Green's Fractures in Adults, ed 6. Philadelphia, PA, Lippincott Williams and Wilkins, 2006, pp 2249-2291 Beaty JH, Kasser JR (eds): Rockwood and Green’s Fractures in Children, ed 6. Philadelphia, PA, Lippin- cott Williams and Wilkins, 2006, pp 1129-1180. 2010 Pediatrie Orthopaedic Examination Answer Book * 23 Question 19 ‘The clinical factors shown to most significantly predict the long-term outcome of Perthes disease of the hip include which of the following? Limb-length discrepancy, range of motion of the hip Age at presentation, range of motion of the hip Age at presentation, limb-length discrepancy Range of motion, pain/limp for more than 6 months Limb-length discrepancy, pain/limp for more than 6 months yeepo PREFERRED RESPONSE: 2 DISCUSSION: Age at presentation and range of motion of the hip are the two most significant predictors of long-term outcome, Younger patients and patients who maintain range of motion of the hip are more likely to have a good outcome, In Herring’s study, children with a chronologic age of younger than & years or a bone age of less than 6 years had significantly more favorable outcomes compared with older children. Limited hip range of motion may be due fo muscle spasm early on, or synovitis; but in late disease, it may reflect incongruity of the joint. Classifications based on femoral head shape have also been correlated to prognosis. Significant shortening of the affected hip is not common. REFERENCES: Herring JA, Kim HT, Browne R: Legg-Calve-Perthes disease. Part II: Prospective multicenter study of the effect of treatment on outcome. J Bone Joint Surg Am 2004:86:2121-2134. Herring JA, Kim HT, Browne R: Legg-Calve-Perthes disease. Part I: Classification of radiographs with use of the modified lateral pillar and Stulberg classifications. J Bone Joint Surg Am 2004;86:2103-2120. Skaggs DL, Tolo VT: Legg-Calve-Perthes disease. J Am Acad Orthop Surg 1996;4:9-16. Question 20 Pediatrie flexor tendon injuries of the upper extremity differ from adult flexor tendon injuries in which of the following ways? 1. Delayed presentation is not common. 2. Astaged repair is never necessary. 3. Six to eight weeks of postoperative immobilization is recommended, 4, Cooperation with occupational therapy can be difficult. 5. The use of Botulinum is contraindicated. PREFERRED RESPONSE: 4 DISCUSSION: Pediatric flexor tendon injuries have several remarkable distinctions from those in adults, Delayed presentation is more common in children, at times requiring staged flexor tendon reconstruction. ‘Three to four weeks of postoperative immobilization following acute repair is recommended in children as opposed to early motion protocols used in adults, Temporary paralytic agents (botulinum toxin type A) have also been shown to facilitate the rehabilitation phase of flexor tendon care in very young children, 24+ American Academy of Orthopsedic Surgeons REFERENCE: Fischgrund JS (ed): Orthopedic Knowledge Update 9, Rosemont, IL, American Academy of Orthopaedic Surgeons, 2008, p 675. Question 21 A 14-year-old boy is seen for back pain. Radiographic evaluation reveals a grade Il isthmie spondylolisthesis. What measurement is most useful in predicting the likelihood of progression? Pelvic incidence Slip angle Sacral inclination Lumbosacral joint angle Sagittal rotation veers PREFERRED RESPONSE: 2 DISCUSSION: Slip angle has been shown to be highly predictive of the risk for increased slippage in patients with spondylolisthesis. None of the other radiographic parameters listed has been shown to be predictive of the risk for increased slippage. REFERENCES: Huang RP, Bohlman HH, Thompson GH, et al: Predictive value of pelvic incidence in progression of spondylolisthesis. Spine 2003;28:2381-2385, Mag-Thiong JM, Wang Z, de Guise JA, et al: Postural model of sagittal spino-pelvic alignment and its relevance for lumbosacral developmental spondylolisthesis. Spine 2008;33:2316-2325. 2010 Pediatric Orthopaedic Examination Answer Book + 25 Figure 22a Figure 22b Question 22 An 8-year-old girl has asymmetry on a forward bend test of the spine. She is asymptomatic and has a normal clinical neurologic examination, Radiographs are shown in Figures 22a and 22b. What should be the next step in her work-up? MRI of the cervical thoracic lumbar spine Supine side bending radiographs of the spine Return to the clinic in 12 months with repeat radiographs Anterior and posterior spinal fusion with instrumentation Echocardiogram and renal ultrasound yaere PREFERRED RESPONSE: 1 DISCUSSION: There are several reasons to obtain an MRI of the entire spinal cord of this patient to evaluate for abnormalities. These include her young age and the presence of a left-sided curve. For juvenile scoliosis patients with more than a 20-degree Cobb angle, there is an approximately 20% prevalence of a neurologic abnormality. Therefore, recommendations for work-up include an MRI scan of the entire spine. REFERENCES: Gillingham BL, Fan RA, Akbarnia BA: Early onset idiopathic scoliosis. J Am Acad Orthop Surg 2006; 14: 101-112. Gupta P, Lenke LG, Bridwell KH: Incidence of neural axis abnormalities in infantile and juvenile patients with spinal deformity: Is a magnetic resonance image screening necessary? Spine 1998;23:206-210. 26+ American Academy of Orthopaedic Surgeons Figure 232 Figure 23b Question 23 A 10-year-old girl returns for follow-up of a right Salter II distal radius fracture she sustained a year ago. She reports pain and increasing deformity of her wrist. A radiograph and clinical photograph are shown in Figures 23a and 23b. What is the next step in management? CT scan to evaluate the extent of the growth arrest Osteotomy of the radius and epiphysiodesis of the ulna Physical therapy and further follow-up Osteotomy of the radius and ulna Bilateral epiphysiodesis of the radius and ulna veers PREFERRED RESPONSE: 1 DISCUSSION: The radiograph and clinical photograph show a growth arrest of the distal radius on the right. There is shortening and narrowing of the physis of the radius, and there is radial deviation of the hand. Greater than 2 cm of growth still remains in the distal radius of a 10-year-old girl. Epiphysiodesis, of both bones bilaterally would leave the same deformity. The first step in treatment is to evaluate the extent of the growth arrest to see if the arrest is resectable. Lengthening of the radius and epiphysiodesis of the ulna could restore the proper length and alignment and would be the treatment of choice if the arrest ‘was not resectable. Osteotomy of the radius and ulna would not address the growth disturbance REFERENCES: Pritchett JW: Growth and development of the distal radius and ulna, J Pediatr Orthop 1996; 16:575-577 Waters PM, Bae DS, Montgomery KD: Surgical management of posttraumatic distal radial growth arrest in adolescents, J Pediatr Orthop 2002;22:717-724. 2010 Pediatric Orthopaedic Examination Answer Book + 27 Figure 24 Question 24 ‘A4-year-old girl has knee pain after a fall. Examination reveals tendemess about the proximal tibia with modest deformity. She has no neurovascular deficits. A radiograph is seen in Figure 24. What should her parents be told? The injury will do well with treatment in a knee immobilizer. The injury will do well with immobilization in a long leg cast. Long leg cast immobilization and reduction is recommended but she may develop a deformity that always requires surgical correction, 4. Long leg cast immobilization and reduction is recommended but she may develop 2 deformity; however, if the deformity develops there is a significant chance it will resolve spontancously. 5. Surgery is indicated to prevent deformity that may follow with this injury. PREFERRED RESPONSE: 4 DISCUSSION: The patient has a so-called Cozen fracture, and she is at significant risk for a posttraumatic genu valgum deformity. However, long-term studies have shown that when such a deformity occurs, it frequently resolves spontaneously and therefore surgical intervention to try and prevent the deformity is not advised REFERENCES: Jordan SE, Alonso JE, Cook FF: The etiology of valgus angulation after metaphyseal fractures of the tibia in children. J Pediatr Orthop 1987;7:450-457 Tuten HR, Keeler KA, Gabos PG, et al: Posttraumatic tibia valga in children: A long-term follow-up note. J Bone Joint Surg Am 1999;81:799-810. 28 + American Academy of Orthopaedic Surgeons Question 25 ‘You are asked to consult on a 4-day-old neonate admitted because of failure to thrive. She has swelling of her left shoulder, Examination reveals limited motion of her hips and left shoulder. Radiographs of the shoulder and pelvis are negative. Laboratory studies show a WBC count of 24,000/mm3, an erythrocyte sedimentation rate of 50/h, and C-reactive protein is 16.4. What is the next most appropriate step in management? Ultrasound of the hip and shoulder Bone scan MRI of the shoulder Pavlik harness Excision and drainage weene PREFERRED RESPONSE: | DISCUSSION: Ultrasound of both the hip and the shoulder can show the presence of septic arthritis and osteomyelitis, Multiple sites of infection are common in neonates. A bone scan can be used to identify other areas of involvement. REFERENCES: Wong M, Isaacs D, Howman-Giles R, et al: Clinical and diagnostic features of osteomyelitis occurring in the first three months of life. Pediatr Infect Dis J 1995;14:1047-1053. Abel MF (ed): Orthopaedic Knowledge Update: Pediatrics 3. Rosemont, IL, American Academy of Orthopaedic Surgeons, 2006, pp 57-73 Question 26 Which of the following statements best describes the prognosis following a fracture of the distal femoral physis? There is a high risk of nonunion. There is a high risk of premature growth arrest that frequently causes deformity. ‘There is a high risk of premature growth arrest but it rarely causes deformity. There is a low risk of premature growth arrest but when it occurs it usually causes deformity. There is a low risk of premature growth arrest and when it occurs it rarely causes deformity. yee PREFERRED RESPONSE: 2 DISCUSSION: Displaced physeal fractures of the distal femur are at high risk for causing premature growth arrest of the involved physis and subsequent deformity. Nonunion of these fractures is extremely rare, REFERENCES: Arkader A, Warner WC Jr, Horn BD, et al: Predicting the outcome of physeal fractures of the distal femur. J Pediatr Orthop 2007:27:703-708. ‘Thomson JD, Stricker SJ, Williams MM: Fractures of the distal femoral epiphyseal plate. J Pediatr Orthop 1995;15:474-478. 2010 Pediatrie Orthopaedic Examination Answer Book * 29 Figure 27a Figure 27b Question 27 Four days ago, a 13-year-old boy stubbed his toe on a chair while running barefoot through his home. He received no treatment at the time. He is now seen at the orthopaedic clinic with the radiograph and clinical photograph shown in Figures 27a and 27b, What is the next step in management? 1. Buddy taping to the adjacent toe and use of a hard-soled shoe for 2 weeks Buddy taping to the adjacent toe and use of a cast extending to the tips of the toes for 3 weeks 3. Open reduction and internal fixation of the fracture, with irrigation of the wound and postoperative antibiotics 4, Antibiotics and closed treatment of the fracture 5. Closed pinning of the phalanx fracture PREFERRED RESPONSE: 3 DISCUSSION: The boy has a Seymour’s fracture of the toe, The germinal matrix of the nail bed is trapped in the fracture site; thus this should be considered an open-fracture. Ideally, it should be treated with open reduction and internal fixation and use of antibiotics at the time of injury. Because this is a delayed presentation, it is even more important to do a formal open reduction and a good irrigation and debridement, followed by the use of postoperative antibiotics. Because the fracture has been displaced for several days, overall management will be easier if the fracture reduction is maintained with pin fixation. REFERENCE: Fischgrund JS (ed): Orthopedic Knowledge Update 9. Rosemont, IL, American Academy of Orthopaedic Surgeons, 2008, p 737. Question 28 A healthy 2-year-old boy falls from a swing and sustains a displaced midshaft femoral fracture with 1 cm of shortening. What is the most appropriate treatment? Pavlik harness Skeletal traction for 3 weeks followed by a spica cast Skin traction for 3 weeks followed by a spica cast Closed reduction and spica casting Closed reduction and an intramedullary pin yeepe PREFERRED RESPONSE: 4 30+ American Academy of Orthopaedic Surgeons DISCUSSION: For children between the ages of 1 and 6 years, closed reduction and early spica casting is recommended. In some instances, associated injuries or body habitus may preclude cast treatment. Pavlik harness treatment of femoral fractures is for infants younger than 1 year of age. Rarely is there an indication for traction. Internal fixation is reserved in general for children older than age 6 years or with confounding factors. REFERENCES: Abel MF (ed): Orthopaedic Knowledge Update: Pediatrics 3. Rosemont, IL, American Academy of Orthopaedic Surgeons, 2006, pp 271-280. Flynn JM, Schwend RM: Management of pediatric femoral shaft fractures. J Am Acad Orthop Surg 2004;12:347-359, Figure 29a Figure 29b Question 29 ‘A 10-year-old boy hit a tree with his sled and is seen in the emergency department with extreme left hip pain and inability to ambulate. He has no history of pain in the left groin, thigh, or knee. Radiographs are seen in Figures 29a and 29. What is the most common complication resulting from this injury? 1. Femoral artery intimal tear 2. Femoral nerve injury 3. Nonunion 4. Malunion 5. Osteonecrosis of the femoral head PREFERRED RESPONSE: 5 DISCUSSION: The child has a type [ hip fracture without associated dislocation. This is an acute hip fracture through the proximal femoral physis, and can occur with or without associated dislocation. He had no prodrome of hip or thigh pain and no femoral neck changes to indicate that this is an unstable slipped capital femoral epiphysis. Osteonecrosis in these transepiphyseal hip fractures is the most common and most devastating complication. The rate of osteonecrosis is most dependent on the initial displacement of the fracture. These fractures should be treated emergently, and decompression of the hip joint is recommended by many authors. 2010 Pediatrie Orthopaedic Examination Answer Book + 31 REFERENCES: Moon ES, Mehlman CT: Risk factors for avascular necrosis after femoral neck fractures in children: 25 Cincinnati cases and meta analysis of 360 cases. J Orthop Trauma 2006;20:323-329. Canale ST: Fractures of the hip in children and adolescents, Orthop Clin North Am 1990;21:341-352, Question 30 A 10-day-old girl has decreased active motion of the left upper extremity. The mother reports a difficult vaginal delivery with presumed shoulder dystocia. Examination shows full passive range of motion of the shoulder, elbow, and wrist but only active flexion of the fingers and wrist. Factors predictive of a good ‘outcome include which of the following? Breech delivery Absence of an ipsilateral clavicle fracture Homer's sign and an APGAR score of 10 at 1 minute Return of active biceps before 3 months and preservation of full passive shoulder range of motion 5. Absent Moro and Babinski reflexes ae PREFERRED RESPONSE: 4 DISCUSSION: Return of active biceps before 3 months and preservation of full passive shoulder range of motion are predictors of a good outcome. Breech delivery is usually associated with preganglionic injury. Preganglionic injury can result in a Homer's sign, which includes ptosis, myosis, and anhydrosis, Preganglionic injuries are unlikely to recover. The Moro reflex is elicited by dropping a baby’s head a short distance and observing active elbow extension and fanning of the fingers, followed by elbow flexion and crying. Absence of the Moro reflex suggests a poor prognosis. REFERENCES: Smith NC, Rowan P, Benson LJ, et al: Neonatal brachial plexus palsy: Outcome of absent biceps function at three months of age. J Bone Joint Surg Am 2004;86:2163-2170. ‘Waters PM: Obstetric brachial plexus injuries: Evaluation and management. J Am Acad Orthop Surg 1997;5:205-214. 32+ American Academy of Orthopaedic Surgeons Figure 31 Question 31 A9-month-old nonambulatory gitl is seen in the emergency department with a fracture of her right forearm. The mother says she fell from the changing table yesterday and continues to ery and not use her right arm. Radiographs are shown in Figure 31. Treatment should consist of which of the following? 1. Closed reduction and a long arm cast 2. Closed reduction, a long arm cast, and a skeletal suryey 3. Closed reduction, a long arm cast, a skeletal survey, and a referral to child protective services 4. Closed reduction and a long arm cast, a bone scan, and referral to child protective services 5. Closed reduction and a long arm cast, MRI of the brain, and a referral to child protective services PREFERRED RESPONSE: 3 DISCUSSION: The occurrence of a forearm fracture in a 9-month-old child has a greater than 50% chance that the injury is due to child abuse. It is mandatory to report this to child protective services unless there is some compelling reason that it is definitely not child abuse. In addition, a skeletal survey should be requested to look for other injuries. A bone scan would show other injuries, but a skeletal survey is a more efficient way to evaluate for other fractures. A MRI of the brain is not indicated unless fundoscopic examination reveals an abnormality. REFERENCES: Kocher MS, Kasser JR: Orthopaedic aspects of child abuse. J Am Acad Orthop Surg 2000;8:10-20. Chang DC, Knight V, Ziegfeld S, et al: The tip of the iceberg for child abuse: The critical roles of the pediatric trauma service and its registry. J Trauma 2004;57;1189-1198, 2010 Pediatric Orthopaedic Examination Answer Book + 33 Question 32 ‘A child with an idiopathic clubfoot is successfully treated by the Ponseti method. The tisk of recurrence of the deformity is most dependent on which of the following factors? Maternal age Positive family history Family’s compliance with bracing The child’s age at walking The child’s body mass index payee PREFERRED RESPONSE: 3 DISCUSSION: The recurrence rate of clubfoot deformity after successful correction by the Ponseti method has been shown to inversely correlate with reported brace compliance. Maternal age, walking age, and body mass index have not been correlated to recurrence. A positive family history increases the risk of a child being bom with a clubfoot but does not influence the recurrence rate. REFERENCES: Dobbs MB, Rudzki JR, Purcell DB, et al: Factors predictive of outcome after use of the Ponseti method for the treatment of idiopathic clubfeet. J Bone Joint Surg Am 2004;86:22-27, Noonan KJ, Richards BS: Nonsurgical management of idiopathic clubfoot. J Am Acad Orthop Surg 2003;11:392-402. " Figure 33 Question 33 An 18-month-old child was involved in a motor vehicle accident and sustained an isolated injury to the left upper extremity. A radiograph is shown in Figure 33. What is the most appropriate management for this injury? 1 z: 3 4, Hanging arm cast Closed reduction with flexible intramedullary nail fixation Coaptation splinting and bandaging the arm to the thorax Closed reduction and external fixation Locking plate fixation PREFERRED RESPONSE: 3 DISCUSSION: Humeral shaft fractures in infants and young children heal rapidly and have excellent remodeling potential. Appropriate treatment in this age group is immobilization with a coaptation splint and bandaging the arm to the thorax for comfort, Internal fixation is appropriate in multiple trauma, and external fixation may be useful when soft-tissue injury is extensive. 34+ American Academy of Orthopaedic Surgeons REFERENCES: Caviglia H, Garrido CP, Palazzi FF, et al: Pediatric fractures of the humerus, Clin Orthop Relat Res 2005;432:49-56, ‘Husain SN, King EC, Young JL, et al: Remodeling of birth fractures of the humeral diaphysis. J Pediatr Orthop 2008;28:10-13. a Figure 34 Question 34 A7-yeat-old girl with a known diagnosis of neurofibromatosis has neck pain and deformity, She has been wearing a soft cervical collar for the past 2 months with mild relief of her symptoms. An MRI scan shows several small neurofibromas on the left side of the cervical spine near the foramina at C6 and 7. A lateral cervical spine radiograph is shown in Figure 34. What is the most appropriate management? Anterior and posterior spinal fu: Anterior spinal fusion In situ posterior fusion Halo traction correction and posterior fusion Continued soft cervical collar treatment yee PREFERRED RESPONSE: 1 DISCUSSION: With a diagnosis of neurofibromatosis and severe kyphosis, anterior and posterior treatment is needed to achieve correction and fusion. In situ fusion has a high failure rate with the kyphotic deformity and even with traction, correction of the kyphosis is not expected. Anterior treatment alone may achieve correction, but in neurofibromatosis only circumferential treatment has been shown to provide long-term stability, REFERENCES: Crawford AH, Schorry EK: Neurofibromatosis update, J Pediatr Orthop 2006;26:413- 423. Mehiman CT, Al-Sayyad MJ, Crawford AH: Bffectiveness of spinal release and halo-femoral traction in ‘the management of severe spinal deformity, J Pediatr Orthop 2004;24:667-673. 2010 Pediatric Orthopaedic Examination Answer Book * 35, Figure 35, Question 35 A 10-month-old infant has no flexion at the elbows, mild flexion contractures at the wrist, a rigid clubfoot deformity on the left foot, and a rigid rocker bottom deformity on the right foot. Examination of the patient's hips reveals limited abduction with 80 degrees of hip flexion/extension and full range of motion of the knees. A radiograph of the pelvis is seen in Figure 35, What is the most appropriate treatment for the patient's hip problem? Preliminary skin traction followed by closed reduction under general anesthesia Immediate closed reduction under anesthesia Preliminary skeletal traction followed by closed reduction under general anesthesia Bilateral open reduction performed through a medial approach Bilateral open reduction performed through an anterior approach peers PREFERRED RESPONSE: 4 DISCUSSION: The patient has arthrogryposis. Szoke and associates performed open reduction through a medial approach on 40 hip dislocations in 26 patients with this condition and reported good results in 80% and fair results in 12%, Due to the stiffness associated with this disorder, closed reduction with or without skin or skeletal traction is not feasible. Open reduction through an anterior approach is reserved for older children, REFERENCES: Szoke G, Staheli LT, Jaffe K, et al: Medial-approach open reduction of hip dislocations in amyoplasia-type arthrogryposis. J Pediat Orthop 1996;16:127-130. Staheli LT, Chew DE, Elliott JS, et al: Management of hip dislocations in children with arthrogryposis. J Pediatr Orthop 1987:7:681-685. 36+ American Academy of Orthopaedic Surgeons Figure 36a Figure 36b Question 36 ‘A 10-year-old boy tripped as he was running down a hill, felt a painful pop in his right knee, and was unable to bear weight on the involved lower extremity. Examination reveals a tense effusion and an extensor lag of the right knee. Figures 36a and 36b show AP and lateral radiographs. Management should consist of long leg casting in 30 degrees of flexion for 6 weeks. a long leg cast in full extension for 6 weeks. knee arthroscopy to rule out internal derangement. physical therapy for range of motion and quadriceps strengthening open reduction and internal fixation. peep PREFERRED RESPONSE: 5 DISCUSSION: The examination and radiographs are consistent with a sleeve fracture of the patella, which is an avulsion fracture of the distal pole of the patella with a disruption of the extensor mechanism. Treatment is open reduction and internal fixation of the patella, and repair of the extensor mechanism. The distal fragment can be much larger than it appears on the radiographs because it consists largely of cartilage. REFERENCES: Wu CD, Huang SC, Liu TK: Sleeve fracture of the patella in children: A report of five cases, Am J Sports Med 1991;19:525-528. Grogan DP, Carey TP, Leffers D, et al: Avulsion fractures of the patella, J Pediatr Orthop 1990;10:721- 730. 2010 Pediatric Orthopaedic Examination Answer Book + 37 Question 37 ‘When addressing a proximal intertrochanteric or subtrochanteric fracture in a juvenile with open growth plates, the arterial supply from what artery at the neck must be preserved? Lateral femoral circumflex Medial femoral circumflex Superior gluteal Inferior gluteal Obturator yeep PREFERRED RESPONSE: 2 DISCUSSION: The medial femoral circumflex artery supplies blood to the femoral head. Its position along the posterior-superior femoral neck places this structure at risk with intramedullary nailing of the femur. Therefore, lateral entry through the greater trochanter is preferred when intramedullary fixation is performed. REFERENCES: Gordon JE, Swenning TA, Burd TA, et al: Proximal femoral radiographic changes after lateral transtrochanteric intramedullary nail placement in children. J Bone Joint Surg Am 2003;85:1295- 1301. Green NE, Swiontkowski MF: Skeletal Trauma in Children, ed 3. Philadelphia, PA, WB Saunders, 2003, pp 419-424. Question 38 What is the primary cause of the decreasing incidence of hemophilic arthropathy in the last 10 to 20 years? Aggressive physical therapy of involved joints Selective joint injections with steroids Availability of and use of home factor treatment Surgical debridement of involved joints Use of splinting to prevent joint contractures wapDe PREFERRED RESPONSE: 3 DISCUSSION: Home factor treatment has decreased the incidence of hemophilic arthropathy. Since 1992, recombinant factor VIII was approved in the United States and can be safely used as prophylaxis or episodic treatment of hemarthrosis. Essentially, recombinant factor VITI eliminated the risk of HIV and hepatitis virus transmission associated with plasma-derived coagulation factors. REFERENCES: Luck JV Jr, Silva M, Rodriguez-Merchan EC, et al: Hemophilic arthropathy. J Am Acad Orthop Surg 2004;12:234-245. Manco-Johnson MJ, Abshire TC, Shapiro AD, et al: Prophylaxis versus episodic treatment to prevent joint disease in boys with severe hemophilia. N Engl J Med 2007;357:535-544. 38 + American Academy of Orthopaedic Surgeons Question 39 The use of bisphosphonates in children with osteogenesis imperfecta is becoming more widely accepted as treatment to improve quality of life and to decrease the risks of fracture, What is the mechanism by which bisphosphonates work? Inhibits osteoclasts Stimulates osteoblasts Increases gastrointestinal absorption of calcium Decreases renal excretion of calcium ‘Acts as a transcription factor to increase production of type I collagen yee e PREFERRED RESPONSE: | DISCUSSION: The mechanism by which bisphosphonates act is by inhibiting osteoclasts. One mechanism of bisphosphonates is to cause osteoclast apoptosis. Another mechanism of bisphosphonates is to disrupt the cytoskeleton of osteoclasts, resulting in loss of the ruffied border. The uncoupling of bone resorption and bone formation with decreased bone resorption results in increased bone mineralization. This translates into fewer fractures in patients with osteogenesis imperfecta and improved quality of life. REFERENCES: Burnei G, Vlad C, Georgescu I, et al: Osteogenesis imperfecta: Diagnosis and treatment. J Am Acad Orthop Surg 2008; 16:356-366, Lin JT, Lane JM: Bisphosphonates. J Am Acad Orthop Surg 2003511:1-4. Seikaly MG, Kopanati $, Salhab N, et al: Impact of alendronate on quality of life in children with osteogenesis imperfecta. J Pediatr Orthop 2005;25:786-791. 2010 Pediatric Orthopaedic Examination Answer Book + 39 Figure 40 Question 40 Evaluation of a nonambulatory 11-year-old girl with spinal muscular atrophy reveals mild scoliosis and full painless range of motion in her hips. An AP radiograph of her pelvis is shown in Figure 40, What is the most appropriate management for the hips? Observation Closed reduction and spica cast application Abduction bracing Open reduction and capsulorrhaphy of the hip Total hip arthroplasty PREFERRED RESPONSE: | peepee DISCUSSION: Hip instability in nonambulatory children with spinal muscular atrophy has been shown to be an infrequent cause of pain or disability; therefore, aggressive treatment generally is not indicated. Observation is the most appropriate management. REFERENCES: Sporer SM, Smith BG: Hip dislocation in patients with spinal muscular atrophy. J Pediatr Orthop 2003;123:10-14, Thompson CE, Larsen LJ: Recurrent hip dislocation in intermediate spinal atrophy. J Pediatr Orthop 1990;10:638-641. Question 41 ‘The addition of which of the following food supplements may lead to a decrease in neural tube defects? Vitamin D-1,25 Vitamin B-12 Niacin Folic Acid Thiamine yeepe PREFERRED RESPONSE: 4 DISCUSSION: The use of folic acid in developed countries has lead to a decrease in neural tube defects, ‘The incidence of neural tube defects is increased in third world countries. 40 + American Academy of Orthopaedic Surgeons REFERENCES: Abel MF (ed): Orthopaedic Knowledge Update: Pediatrics 3. Rosemont, IL, American Academy of Orthopaedic Surgeons, 2006, pp 111-122. Lemke L, Dias L: Spina bifida, in Cramer KE, Scherl SA, Einhorn TA (eds): Orthopaedic Surgery Essentials: Pediatrics, Philadelphia, PA, Lippincott Williams & Wilkins, 2004, pp 203-210. Figure 42 Question 42 A S-year-old boy had a 1-week history of left hip pain and a limp that resolved 5 weeks prior to his office visit. Examination demonstrates a pain-free and symmetric range of motion. A radiograph is seen in Figure 42. What is the next step in management? Physical therapy for range of motion and strengthening of the hips Hip abduction brace wear Left Salter pelvie osteotomy Limitations of activities and observation Radiographs of the knees and spine peeve PREFERRED RESPONSE: 5 DISCUSSION: Whereas bilateral Perthes of the hips occurs in 11% of cases, in patients with symmetric changes/stages, other diagnoses must be considered such as Meyers dysplasia. Multiple epiphyseal dysplasia is most readily diagnosed by evaluation of other radiographs, in particular of the knee and, if confirmatory, of the spine to assess for spondyloepiphyseal dysplasia. REFERENCES: Herring JA (ed): Tachdjian’s Pediatric Orthopaedics, ed 4, Philadelphia, PA, WB Saunders, 2008, pp 806-810. Hesse B, Kohler G: Does it always have to be Perthes’ disease? What is epiphyseal dysplasia? Clin Orthop Relat Res 2003;414:219-227, 2010 Pediatric Orthopaedic Examination Answer Book + 41 Question 43 Which of the following is a characteristic of odontoid fractures in children? Usually occur in the body of C2 Are reduced by gentle cervical fiexion Frequently progress to nonunion ‘Almost always occur at the basilar synchondrosis ‘Are commonly associated with neurologic injury peepee PREFERRED RESPONSE: 4 DISCUSSION: Fracture of the odontoid process in children is usually caused by a fall, motor vehicle accident, or minor trauma, and almost always occurs through the synchondrosis at the base of the dens. Neurologic deficits are rare in isolated odontoid fractures in children. Closed reduction by neck extension and immobilization using a cast, a brace, or halo traction for 6 to 8 weeks is usually sufficient to allow the fracture to heal. REFERENCE: Fischgrund JS (ed): Orthopedic Knowledge Update 9. Rosemont, IL, American Academy of Orthopaedic Surgeons, 2008, p 692. Question 44 ‘A 6-year-old boy is being treated for acute hematogeneous osteomyelitis of the distal femur with intravenous antibiotics. The best method to determine the success or failure of initial treatment is by serial evaluations of which of the following studies? 1. Radiographs 2. MRI 3. Erythrocyte sedimentation rate (ESR) 4. CBC with differential 5. C-reaetive protein (CRP) PREFERRED RESPONSE: 5 DISCUSSION: Successful antibiotic treatment of acute osteomyelitis should lead to a rapid decline in the CRP. The CRP is the most sensitive study to follow the treatment of osteomyelitis. The CRP should decline after 48 to 72 hours of appropriate treatment. CBC and ESR are helpful in initial evaluation and diagnosis, but remain abnormal in the early phase of treatment regardless of response. Imaging studies are useful for surgical planning or secondarily if the CRP remains elevated REFERENCES: Unkila-Kallio L, Kallio MJ, Eskola J, et al: Serum C-reactive protein, erythrocyte sedimentation rate, and white blood cell count in acute hematogenous osteomyelitis of children, Pediatrics 1994;93:59-62, Herring JA(ed): Tachdjian’s Pediatric Orthopaedics, ed 4. Philadelphia, PA, WB Saunders, 2008, pp 2090-21100, 42+ American Academy of Orthopaedic Surgeons Question 45 What is the most important predictor of functional outcome in patients with myelomeningocele? 1. Functional motor level 2. Sensory level 3. Dysplasia of the hip 4. Foot deformity 5. Hydrocephalus PREFERRED RESPONSE: 1 DISCUSSION: The functional motor level of the patient is of prime importance in determining prognosis and outcome. Patients with thoracic and upper lumbar motor levels will need wheelchairs or hip-knee- ankle-foot orthoses to ambulate at all. Patients with midlumbar motor levels can be household or limited ‘community walkers, whereas children with low lumbar or sacral motor levels are likely to be able to walk in the community. REFERENCES: Abel MF (ed): Orthopaedic Knowledge Update: Pediatrics 3. Rosemont, IL, American Academy of Orthopaedic Surgeons, 2006, pp 117-120. ‘Swank M, Dias L: Myelomeningocele: A review of the orthopaedic aspects of 206 patients treated from birth with no selection criteria. Dev Med Child Neurol 1992;34:1047-1052. Figure 46a Figure 46b Question 46 A 12-year-old boy reports a 6-week history of left hip pain, He denies any history of trauma or fever. Examination reveals diminished internal rotation of both hips and discomfort with this manuever. Radiographs are shown in Figures 46a and 46b. What is the most appropriate management? Surgical in situ pinning of the left hip Surgical dislocation with reduction of the left slipped capital femoral epiphysis, In situ pinning of bilateral hips Bed rest Application of a hip spica cast veep PREFERRED RESPONSE: 3 2010 Pediatric Orthopaedic Examination Answer Book + 43 DISCUSSION: The patient has left hip pain and clinical and radiographic evidence of a left slipped capital femoral epiphysis, He also has open triradiate cartilage and a grade | slip on the right side that, at the present time, is silent, ‘The best treatment is pinning of bilateral slipped capital femoral epiphysis. Reduction is not indicated because of the mild nature of both slips. Although prophylactic pinning of the uninvolved contralateral hip is controversial, this patient shows a clinically silent grade 1 slip on the right side, REFERENCES: Puylaert D, Dimeglio A, Bentahar T: Staging puberty in slipped capital femoral epiphysis: Importance of the triradiate cartilage. J Pediatr Orthop 2004;24:144-147. Dewnany G, Radford P: Prophylactic contralateral fixation in slipped upper femoral epiphysis: J Pediatr Orthop B 2005;14:429-433. it safe? Question 47 Nutritional rickets in the US occurs more frequently in infants older than 6 months of age who do not receive vitamin D supplementation and are Caucasian and formula fed. Caucasian and breast fed. African American and formula fed. African American and breast fed. Asian and formula fed. peepee PREFERRED RESPONSE: 4 DISCUSSION: Numerous reports suggest an increased frequency of nutritional rickets in the US in children with dark skin pigmentation who are breast fed past 6 months of age without vitamin D supplementation, Nutritional rickets is rare in light-skinned children or those who are formula fed. REFERENCES: Herring JA (ed): Tachdjian’s Pediatric Orthopaedics, ed 4. Philadelphia, PA, WB Saunders, 2008, p 1918. Weisberg P, Scanlon KS, Li R, et al: Nutritional rickets among children in the United States: Review of cases reported between 1986 and 2003. Am J Clin Nutr 2004;80:16978-1705S. Jacobsen ST, Hull CK, Crawford AH; Nutritional rickets. J Pediatr Orthop 1986;6:713-716 44+ American Academy of Orthopaedic Surgeons Question 48 A 10-year-old girl has had nontraumatic swelling of the left knee for the past month, No other joints are swollen and there is no history of fever, although the patient’s mother does recall the child having a localized, but expanding “target-like” rash a few months ago when the family was vacationing in Connecticut, Examination of the knee reveals moderate swelling with no tenderness and near full range of motion, The child lacks perhaps the final 15 degrees of extension and the final 20 degrees of flexion. Laboratory studies show a normal CBC count but the erythrocyte sedimentation rate is 35 mnvh (0-20 normal). Antinuclear antibody test and rheumatoid factor tests are negative, What is the most likely diagnosis? 1. Acute rheumatic fever 2. Septic arthritis 3. Transient synovitis 4, Lyme arthritis 5. Gout PREFERRED RESPONSE: 4 DISCUSSION: Lyme arthritis is associated with a tick bite and is endemic to certain areas of Connecticut. The earliest presentation of the disease is manifested by erythema migrans which is the classic expanding rash that occurs at the site of the tick bite and can develop within 1 week to 1 month after exposure. Joint involvement with manifestation of Lyme arthritis can occur months to years after the initial infection, Most patients have single joint involvement with the knee being the most affected site. Lyme arthritis is a low-grade inflammatory synovitis that can present with large and relatively painless joint effusion. The ‘most effective treatment is with a single 4-week course of oral amoxicillin or doxyeycline, REFERENCES: Feder HM Jr: Lyme disease in children. Infect Dis Clin North Am 2008;22:315-326. Gerber MA, Zemel LS, Shapiro ED: Lyme arthritis in children: Clinical epidemiology and long-term outcomes. Pediatrics 1998;102:905-908, Question 49 ‘An 8-year-old girl was treated for a Salter-Harris type I fracture of the right distal femur 2 years ago. She has symmetric knee flexion, extension, and frontal alignment to her contralateral knee. She has a l-em limb-length discrepancy of the femur. She has always been in the SOth percentile for height and her skeletal age matches her chronologic age. She has « complete physeal closure of the right distal femur. What is the expected limb-length discrepancy at maturity’? 1. 3em 2 6em 3. 10cm 4. 14em 5. 18cm PREFERRED RESPONSE: 2 2010 Pediatric Orthopaedic Examination Answer Book + 45 DISCUSSION: The child has a near complete central physeal arrest of the distal fernur. She will develop worsening limb-length discrepancy, She is growing at the average rate for the population. The distal femoral physis grows roughly at a rate of 9 mm/year. Girls finish their growth roughly at 14 years. Thus, at maturity, the uninjured side will be 6.4 cm longer than the injured side. Since she has not developed an angular deformity at this point and her arrest is central, she is unlikely to develop angular deformity in any plane. REFERENCES: Little DG, Nigo L, Aiona MD: Deficiencies of eurrent methods for the timing of epiphysiodesis. J Pediatr Orthop 1996;16:173-179. Moseley CF: Assessment and prediction in leg-length discrepancy. Instr Course Lect 1989;38:325-330. Figure 50 Question 50 Figure 50 shows the radiographs of a 3-year-old child who has elbow pain. What is the most appropriate treatment? Collar and cuff for comfort Long arm cast Closed reduction and percutaneous pinning Closed reduction and long arm cast Open reduction and internal fixation yReNo PREFERRED RESPONSE: 3 DISCUSSION: The radiographs show an extension type II supracondylar fracture with hyperextension of the distal fragment. There is medial impaction of the fracture as well. If the fracture heals in this alignment, the result will be cubitus varus with a loss of flexion of the elbow. Management should consist of closed reduction that potentially converts the fracture to an unstable fracture. Percutaneous pinning is recommended as cast treatment alone could lead to loss of reduction. ‘Treatment in a cast with hyperflexion may lead to compartment syndrome, 46 American Academy of Orthopaedic Surgeons REFERENCES: Mehserle WL, Meehan PL: Treatment of displaced supracondylar fractures of the humerus (type III) with closed reduction and percutaneous cross-pin fixation. J Pediatr Orthop 1991;11:705-711. Herring JA: Upper extremity injuries: Supracondylar fractures of the humerus, in Herring JA (ed): Tachdjian’s Pediatric Orthopaedics, ed 4. Philadelphia, PA, WB Saunders, 2008, pp 2470-2471. Question 51 ‘An 11-year-old gir! is struck in the leg by a loaded sled while sledding and is seen in the emergency department; she is reporting severe knee pain, Radiographs are read as normal. Examination reveals that she is exquisitely tender over the proximal tibial physis. The neurovascular examination is normal. What is the next step in management? Splinting, admission, and frequent neurovascular checks Cylinder cast and discharge Emergent knee arthroscopy Four-compartment calf fasciotomy Non-weight-bearing, a knee immobilizer, and follow-up in | week yaene PREFERRED RESPONSE: | DISCUSSION: The anatomic lesion in this patient is not exactly defined, but she has most likely sustained an injury about the knee, A Salter-Harris type I proximal tibial physeal fracture is likely. ‘The normal radiograph reading can be misleading because these injuries may displace and spontaneously reduce. The child is at risk of compartment syndrome although she is currently not displaying signs of it, Thus, even though this injury may seem trivial by radiographic findings, it should be treated like a knee dislocation with a risk of late developing compartment syndrome. MRI or CT may be necessary to define the injury. She does not require emergent treatment, but merits close observation for possible compartment syndrome. Any of the possible injuries about the knee can be unstable and require internal fixation after teduction. REFERENCES: McGuigan JA, O’Reilly MJ, Nixon JR: Popliteal arterial thrombosis resulting from disruption of the upper tibial epiphysis. Injury 1984;16:49-50. Burkhart SS, Peterson HA: Fractures of the proximal tibial epiphysis. J Bone Joint Surg Am 1979;61:996- 1002, 2010 Pediatric Orthopaedic Examination Answer Book + 47 Question 52 An otherwise healthy 4-week-old girl is noted on examination of the left hip to have a positive Ortolani and Barlow test. She is placed ina Pavlik harness and returns for interval adjustments. At 3 weeks she returns for a hamess check and an ultrasound reveals a persistent hip dislocation, What is the next most appropriate step in management? Adjustment of the hamess to maintain 80 degrees of abduction Removal of the harness to avoid creating further deformity of the acetabulum Removal of the harness and acceptance of the hip position without further treatment Surgical open reduction of the hip within 2 weeks Continued use of the harness and recheck in 2 to 3 weeks peers PREFERRED RESPONSE: 2 DISCUSSION: The patient has failed to respond to Pavlik harness treatment. If use of the Pavlik hamess fails to maintain reduction at 2 weeks, use of the hamess should be discontinued to avoid creating further deformity of the acetabulum, Altemative treatments considered later include bracing, closed reduction, and spica casting, or open reduction and spica casting. With a Pavlik hamess, continued abduction and hip flexion of the displaced hip may lead to posterolateral acetabular dysplasia. REFERENCES: Guille JT, Pizzutillo PD, MacEwen GD: Development dysplasia of the hip from birth to six months. J Am Acad Orthop Surg 2000;8:232-242. Hedequist D, Kasser J, Emans J: Use of an abduction brace for developmental dysplasia of the hip after failure of Pavlik harness use. J Pediatr Orthop 2003;23:175-177. 48 + American Academy of Orthopaedic Surgeons, Figure 53a Figure 53b Figure 53c Figure 53d Question 53 Figures 53a through 53d show the clinical photographs and radiographs of the lower extremity of a newborn male, Examination reveals this to be an isolated finding. The child otherwise has a normal neurologic examination. The hips are stable and there are no spinal defects. What is the most appropriate ‘treatment at this time? 1. Symes amputation once ambulatory 2. Observation as the deformity will slowly resolve and the child will be left with a limb- length discrepancy 3. Immediate osteotomy for correction of the deformity 4, Casting for correction of the deformity 5. Genetic testing for neurofibromatosis PREFERRED RESPONSE: 2 DISCUSSION: The radiographs and clinical photographs reveal a child with posteromedial bowing of the tibia. This is a congenital anomaly that is associated with a calcaneal valgus foot. It is a relatively benign condition. The severity of the bow diminishes with time; however, the child will be left with a limb-length discrepancy, usually in the range of 4 cm. The residual limb-length discrepancy presents the greatest challenge for orthopaedic management. This, however, can usually be handled with limb-lengthening techniques. Casting can be used for severe cases with unresolving significant contracture; however, gradual spontaneous correction is usually the norm. This condition is quite different from anterior lateral bowing that can be associated with neurofibromatosis and pathologic fracture or pseudoarthrosis of the tibia. REFERENCES: De Maio F, Corsi A, Roggini M, et al: Congenital unilateral posteromedial bowing of the tibia and fibula: Insights regarding pathogenesis from prenatal pathology. A case report. J Bone Joint Surg Am 2005;87:1601-1605. Schoenecker PL, Rich MM: The lower extremity in pediatric orthopaedics, in Morrissy RT, Weinstein SL (eds): Lovell and Winter’s Pediatrie Orthopaedics, ed 6. Philadelphia, PA, Lippincott, Williams and Wilkins, 2006, pp 1198-1200. 2010 Pediatric Orthopaedic Examination Answer Book + 49 sonal Figure 54a Figure 54b Question 54 ‘Anewbom male child has a left foot deformity as shown in Figures 54a and 54b, The family history and birth history are unremarkable, The child is healthy and thriving, and examination of the spine, hips, and neurologic system reveals normal findings. What is the best treatment for the foot deformity? Stretching by the parents Ankle-foot orthosis (AFO) and night splints Anterior tibial tendon transfer Casting with the Ponseti method Short leg cast application peer PREFERRED RESPONSE: 4 DISCUSSION: The foot shows all the classic signs of a clubfoot with hindfoot equinus, heel varus, supination, and forefoot adduction. The Ponseti method is now well recognized as the best treatment for idiopathic clubfoot. It calls for manipulation of the clubfoot on a weekly basis with the application of long, leg cast to slowly achieve correction. A percutaneous heel cord tenotomy is often required, followed by an additional 3-week period of casting and eventual use of a foot abduction orthosis. APO night splints will not achieve any correction. Anterior tibial tendon transfer is sometimes performed for a clubfoot with recurrence or if there is supination in the swing phase of gait. Short leg casts are not sufficient to achieve full correction of a clubfoot. REFERENCES: Herzenberg JE, Radler C, Bor N: Ponseti versus traditional methods of casting for idiopathic clubfoot. J Pediatr Orthop 2002;22:517-521. Morcuende JA, Dolan LA, Dietz FR, et al: Radical reduction in the rate of extensive corrective surgery for clubfoot using the Ponseti method. Pediatrics 2004;113:376-380. 50 + American Academy of Orthopaedic Surgeons Question 55 In Ewing’s sarcoma, neoplastic properties are thought to be related to a environmental toxins, a prior history of osteomyelitis, | a prior history of viral illness, a prior history of trauma. translocation of chromosomes. payee PREFERRED RESPONSE: 5 DISCUSSION: In 95% of patients with Ewing’s sarcoma, there is a translocation, t(11:22). This results in EWS/FLI-] transcription factor that results in tumor cell proliferation. Other mechanisms causing tumor cell proliferation include inactivation of tumor suppressor genes, or activation of proto-oncogenes. REFERENCES: Amdt CA, Crist WM: Common musculoskeletal tumors of childhood and adolescence, 'N Engl J Med 1999;341:342-352. Pierz KA, Womer RB, Dormans JP: Pediatric bone tumors: Osteosarcoma Ewing's sarcoma, and chondrosarcoma associated with multiple hereditary osteochondromatosis. J Pediatr Orthop 2001;21:412- 418. Question 56 Which of the following statements best describes what treatment is required for children with adolescent tibia vara? 1. No treatment is necessary because spontaneous resolution is common. 2. Orthotic treatment is highly effective, 3. Inaddition to proximal tibial osteotomy, valgus deformity of the distal femur quently develops to compensate for the proximal tibia vara and must be corrected with femoral osteotomy. 4. In addition to proximal tibial osteotomy, varus deformity of the distal femur is frequent and must be corrected by distal femoral osteotomy. 5. Surgical elevation of the medial tibial plateau is an integral part of all surgical interventions for this condition. PREFERRED RESPONSE: 4 ' DISCUSSION: Spontaneous resolution of adolescent tibia vara is uncommon. Orthotic treatment has not been shown to be effective. Sungical elevation of the medial tibial plateau is a procedure that is ! occasionally necessary in individuals with early onset Blount’s disease but is not indicated for individuals with late onset Blount’s disease, Distal femoral varus deformity is commonly present and must be addressed. 2010 Pediatric Orthopaedic Examination Answer Book + 51 REFERENCES: Gordon JE, King DJ, Luhmann SJ, et al: Femoral deformity in tibia vara. J Bone Joint Surg Am 2006;88:380-386. Gordon JE, Heidenreich FP, Carpenter CJ, et al: Comprehensive treatment of late-onset tibia vara. J Bone Joint Surg Am 2005;87:1561-1570, Question 57 ‘The parents of a 14-year-old female soccer player are concerned about any future injury. They have been adyised that she has the potential to play for the US Olympic team. ‘They are especially concemed about the anterior cruciate ligament (ACL). What should you advise them? ACL injuries are more common in men younger than 30 years of age. ACL injuries are more common in women younger than 30 years of age. ACL injuries are usually the result of contact sports. The incidence of ACL injuries can be decreased by a neuromuscular training program. ACL injuries are rarely associated with meniscal injury. veepe PREFERRED RESPONSE: 2 DISCUSSION: ACL injuries are five to eight times more common in young women. The highest incidence is associated with basketball and soccer. These sports require rapid directional and rotational changes. Use of neuromuscular training programs has not been associated with a decrease in ACL injuries. It is recommended that there be more frequent rests. ACL injuries are commonly associated with meniscal injury. REFERENCES: Shea KG: ACL Injury: Epidemiology and Prevention Presented at Sports Related Injuries in the Skeletally Mature Athlete. POSNA: One Day Course, 2008. Millett PJ, Willis AA, Warren RF: Associated injuries in pediatric and adolescent anterior cruciate ligament tears: Does a delay in treatment increase the risk of meniscal tears? Arthroscopy 2002;18:955- 959. Question 58 An 8-year-old boy weighing 70 Ib sustains a displaced diaphyseal femur fracture and is treated with two flexible retrograde intramedullary rods, What is the most common complication following treatment with this technique? Limb-length discrepancy Mechanical irritation around the knee Quadriceps weakness Malunion Patellofemoral pain PREFERRED RESPONSE: 2 52+ American Academy of Orthopaedic Surgeons DISCUSSION: Flexible retrograde intramedullary nailing is now the preferred treatment for most length- stable diaphyseal femur fractures in school-aged children, The most commonly described complication is, irritation about the knee at the rod insertion sites that resolves with rod removal. Limb-length discrepancy and weakness have also been described at lower rates. Malunion or rod bending is usually related to placement of the rods in an unstable fracture pattem or in a larger patient. REFERENCES: Flynn JM, Hresko T, Reynolds RA, et al: Titanium elastic nails for pediatric femur fractures: A multicenter study of early results with analysis of complications. J Pediatr Orthop 2001;21:4- 8. Flynn JM, Schwend RM: Management of pediatric femoral shaft fractures. J Am Acad Orthop Surg 2004;12:347-359, Figure 59a Figure 59 Question 59 A 12-year-old girl with foot pain who has been diagnosed with hereditary motor sensory neuropathy is seen for the foot deformity shown in Figure 59a. A “block test” is performed and shown in Figure 59. What is the most appropriate management for this patient? Observation Corrective shoes Plantar release with first metatarsal osteotomy and possible tendon transfers Calcaneal osteotomy Triple arthrodesis, wayne PREFERRED RESPONSE: 3 DISCUSSION: The hindfoot varus in this individual with a cavovarus deformity is nonstructural as shown by the “block test”. Therefore, surgical procedures directed at correcting the hindfoot deformity are not necessary. Observation is not in order and shoe modifications have not been shown to be effective in managing this problem, The patient is symptomatic; therefore, the treatment of choice is plantar release with first metatarsal osteotomy and possible tendon transfers. 2010 Pediatric Orthopaedic Examination Answer Book + $3 REFERENCES: Paulos L, Coleman SS, Samuelson KM: Pes cavovarus: Review of a surgical approach using selective soft-tissue procedures. J Bone Joint Surg Am 1980;62:942-953. McCluskey WP, Lovell WW, Cummings RJ: The cavovarus foot deformity: Etiology and management. Clin Orthop Relat Res 1989;247:27-37. Ward CM, Dolan LA, Bennett DL, et al: Long-term results of reconstruction for treatment of a flexible cavovatus foot in Charcot-Marie-Tooth disease. J Bone Joint Surg Am 2008;90:2631-2642, Figure 60a Figure 60b Figure 60 Question 60 ‘A9-month-old boy fell down three steps onto his elbow. Immediate swelling was noted and he was taken to the emergency department. Radiographs are shown in Figures 60a through 60c. ‘Treatment should consist of 1. asling and early range of motion. a splint in situ, with removal of the splint and range-of-motion exercises in I week. 3. closed reduction in the emergency department, followed by splinting and range-of-motion exercises in 1 week. 4. closed reduction and percutaneous pin fixation in the operating room, with immobilization for 3 to 4 weeks. 5. open reduction and plate fixation in the operating room with early range of motion. PREFERRED RESPONSE: 4 DISCUSSION: The radiographs show a transphyseal fracture separation of the distal humerus. Secondary ossification centers are not yet apparent. The radius and ulna maintain a normal relationship to each other, but not with the humerus. Commonly, as in these radiographs, displacement is medial. Elbow dislocation is more often lateral displacement, These physeal injuries are often stable; however, anatomic reduction and percutaneous pinning is recommended to prevent late deformity. Child abuse can be associated with these fractures. REFERENCES: Flynn JM, Sarwark JF, Waters PM, et al: The surgical management of pediatric fractures of the upper extremity. Instr Course Lect 2003;52:635-645. ‘Oh CW, Park BC, Ihn JC, et al: Fracture separation of the distal humeral epiphysis in children younger than three years old. J Pediatr Orthop 2000;20:173-176. ‘54+ American Academy of Orthopaedic Surgeons "a Figure 61 Question 61 A.220-Ib high school basketball player injured his knee while landing after a rebound. Figure 61 shows a lateral view of the knee. This fracture is associated with which of the following complications? Limb-length diserepeney Varus deformity of the proximal tibia Compartment syndrome Genu procurvatum Ligamentous instability of the knee vaeee PREFERRED RESPONSE: 3 DISCUSSION: There is @ high incidence of compartment syndrome seen in type III tibial tubercle fractures. Fasciotomy should be considered at the time of initial repair. Type III tibial tubercle fractures extending through the joint are often associated with meniscal injuries, which must be repaired. Delayed complications included recurvatum and refracture, Its association with Osgood-Schlatter’s disease has not been proven, This is a fracture that occurs in later adolescence, so significant limb-length discrepencies are unusual after this fracture. REFERENCES: Ogden JA, Tross RB, Murphy MJ: Fracture of the tibial tuberosity in adolescents. J Bone Joint Surg Am 1980;62:205-215. Sponseller PE, Beaty JH: Fractures and dislocations about the knee, in Rockwood CA, Wilkins KE, Beaty JH (eds): Fractures in Children, ed 4. Philadelphia, PA, Lippincott Raven, 1996, pp 1273-1281. 2010 Pediatric Orthopaedic Examination Answer Book + 55 Question 62 In the swing phase of gait, the leg is carried forward by the momentum generated by which of the following? ‘Ankle plantar flexors and hip flexors at terminal stance Knee extensors and ankle dorsiffexors at terminal stance Knee flexors and ankle darsiflexors in early swing, Hip flexors and ankle dorsifiexors in early swing Hip flexors and knee extensors in carly swing payee PREFERRED RESPONSE: | DISCUSSION: More muscle activity occurs during stance phase than during swing phase. During stance phase, the muscles of the leg and foot work to stabilize the plantigrade foot, In swing phase, momentum generated by the gastrocsoleus and hip flexors at terminal stance carries the leg forward. Knee flexion in early swing and then extension at terminal swing occur passively. The main concentric contraction that ‘occurs during swing phase is that of the anterior tibialis, which dorsiflexes the foot for easier clearance during swing and prepositions the foot for initial contact. REFERENCES: Herring JA: Tachdjian’s Pediatric Orthopaedics, ed 4. Philadelphia, PA, WB Saunders, 2008, pp 79-85 ‘Ounpuu S, Gage JR, Davis RB: Three-dimensional lower extremity joint kinetics in normal pediatric gait. J Pediatr Orthop 1991;11:341-349, Kadaba MP, Ramakrishnan HK, Wootten ME: Measurement of lower extremity kinematics during level walking. J Orthop Res 1990;8:383-392. Question 63 Posttraumatic physeal arrest is most common at which of the following locations? Proximal tibia Proximal humerus Distal radius Distal humerus 5. Distal tibia aeN PREFERRED RESPONSE: 5 DISCUSSION: Posttraumatic physeal arrest occurs most commonly in the distal medial tibia. Using MRI, Echlund and associates confirmed this finding. Arrest of the distal radius and proximal humerus are rare after trauma. Traumatic injuries of the distal femoral and distal ulnar physis have a high incidence of growth arrest as well, REFERENCES: Ecklund K, Jaramillo D: Patterns of premature physeal arrest: MR imaging of II children, AJR Am J Roentgenol 2002;178:967-972. Khoshhal KI, Kiefer GN: Physeal bridge resection. J Am Acad Orthop Surg 2005;13:47-58. 56 + American Academy of Orthopaedic Surgeons Figure 64a Figure 64b Question 64 The newborn foot deformity seen in Figures 64a and 64b should initially treated with observation with possible stretching. serial casting. medial surgical release. posterior medial surgical release. dynamic ankle-foot orthosis. yaeye PREFERRED RESPONSE: | DISCUSSION: Mild to moderate metatarsus adductus is best treated with observation and possible passive stretching exercises because most of these feet will self correct. Numerous types of shoes, braces, and splints have been devised but the efficacy of these have not been determined. Serial casting is reserved for severe metatarsus adductus in the infant, although a medial surgical release may be indicated if the deformity is symptomatic and persists beyond age 4 years. REFERENCES: Abel MF (ed): Orthopaedic Knowledge Update: Pediatrics 3. Rosemont, IL, American Academy of Orthopaedic Surgeons, 2006, pp 240-241. Farsetti P, Weinstein SL, Ponseti IV: The Long-term functional and radiographic outcomes of untreated and non-operatively treated metatarsus adductus. J Bone Joint Surg Am 1994;76:257-265. 2010 Pediatric Orthopaedic Examination Answer Book * 57 Question 65 A 4-year-old girl has been limping for the past 2 months, There is no history of trauma, previous injury, fever, or other systemic complaints. Examination reveals a moderate right knee effusion with a 10-degree knee flexion contracture. What is the next most appropriate step in evaluation? Arthroscopy Antinuclear antibody MRI Bone scan HLA-B27 peeps PREFERRED RESPONSE: 2 DISCUSSION: The patient presents with juvenile idiopathic arthritis manifestations. The American College of Rheumatology defines this as one or more joints involved with swelling of 6 weeks or longer. A positive antinuclear antibody test would be diagnostic. Consideration should be made to have the patient see an ophthalmologist for evaluation of possible uveitis, Although the patient could have Lyme disease, that choice is not an option. ‘The presence of an elevated antinuclear antibody by itself should not necessarily be used for diagnosing arthritis; however, the test does have clinical utility as a screening test, The frequency of a positive antinuclear antibody test is greatest in younger girls with oligoarticular disease and carries an increased risk for anterior uveitis. Arthroscopy might be indicated if this patient was presenting with a discoid meniscus, but there is no history of clicking, which is often one of the classic signs of discoid meniscus. MRI would not be used to diagnose juvenile idiopathic arthritis, but MRI would be useful to help diagnose discoid meniscus. A bone scan would show increased uptake in the patient's knee but again, this would not help diagnose her condition. HLA-B27 has no role in diagnosing juvenile idiopathic arthritis, especially in females. REFERENCES: Iesaka K, Kubiak EN, Bong LR, et al: Orthopaedic surgical management of hip and knee involvement in patients with juvenile rheumatoid arthritis. Am J Orthop 2006;35:67-73. Wright DA: Juvenile idiopathic arthritis, in Morrissey RT, Weinstein SL (eds): Lovell and Winter’s Pediatric Orthopaedics, ed 6. Philadelphia PA, Lippincott Williams and Wilkins, 2006, pp 405-438. 58 + American Academy of Orthopaedic Surgeons Question 66 ‘An 18-month-old girl is brought in by her parents because of concerns about intocing, bowlegs, and tripping and falling. Prenatal and birth history are otherwise unremarkable. The child’s growth and development appear to be normal and she has a normal neurologic exam, a straight spine with no defects, and the hips are stable, Examination reveals hip internal rotation of 40 degrees and hip external rotation of 60 degrees. The thigh-foot angle is internal 30 degrees. Feet are straight and supple. Gait is characterized by intoeing with occasional tripping and falling. Based on these findings, what is the most appropriate action? 1. No treatment because internal tibial torsion slowly resolves on its own 2. Immediate treatment with a Denis-Browne bar 3. Distal tibial osteotomies 4. Proximal femoral derotational osteotomies 5. Treatment with twister cables PREFERRED RESPONSE: | DISCUSSION: The child has classic internal tibial torsion that is very commonly seen in younger children ‘who are just beginning to walk. The normal outcome is for slow resolution of this problem and it seldom requires any treatment. Treatment with a Denis-Browne bar or with twister cables has not been proven to be effective. Surgical treatment at this point is premature and clearly not indicated, REFERENCES: Lincoln TL, Suen PW: Common rotational variations in children. J Am Acad Orthop Surg 2003;11:312-320. Staheli LT, Corbett M, Wyss C, et al: Lower-extremity rotational problems in children: Normal values to guide management. J Bone Joint Surg Am 1985;67:39-47. 2010 Pediatric Orthopaedic Examination Answer Book + 59 Figure 67 Question 67 A 5-year-old boy reports intermittent left elbow pain. History reveals that he injured his elbow 4 months ago, but had no treatment. He is now using his arm normally but reports pain almost daily, Examination reveals tendemess over the lateral epicondyle and a prominence is evident. Range of motion is from -5 degrees to 120 degrees, Radiographs are shown in Figure 67. Management should include open reduction and internal fixation. cast immobilization. percutaneous pin fixation. observation, with follow-up in 3 months. an MRI scan of the elbow. weeps PREFERRED RESPONSE: | DISCUSSION: The patient has a nonunion of the lateral condyle of the left humerus. Observation or cast treatment at this stage is not likely to lead to healing of the fracture, MRI will not add any additional information. Open reduction, with minimal posterior soft-tissue stripping, is recommended to establish union of the fracture. Local or other bone graft may also be required. There are no studies that indicate that the displaced fracture will heal with late percutaneous fixation. REFERENCES: Wattenbarger JM, Gerardi J, Johnson CE: Late open reduetion intemal fixation of lateral condyle fractures. J Pediatr Orthop 2002;223:94-398. Flynn JC: Nonunion of slightly displaced fractures of the lateral humeral condyle in children: An update. J Pediatr Orthop 1989;9:691-696. (60 « American Academy of Orthopaedic Surgeons Figure 68 Question 68 A 6-year-old girl sustains an ankle injury after falling on roller blades. An AP radiograph is shown in Figure 68. Treatment should consist of which of the following? Closed manipulation and a long leg cast Closed manipulation and a short leg cast Long leg cast without manipulation Open reduction and internal fixation with a screw crossing the growth plate Open reduction and internal fixation with fixation parallel to the physis paeye PREFERRED RESPONSE: 5 DISCUSSION: The child has a Salter-Harris type IV injury involving both the growth plate and the articular surface of the ankle, ‘This injury pattern has a high risk of physeal arrest; open reduction and internal fixation is indicated to realign the physis and joint surface. ‘The best method of fixation to avoid growth arrest is one that does not cross the physis. This is usually achieved by an epiphyseal screw or pins parallel to the physis. If the metaphyseal fragment were large enough, a transverse metaphyseal screw could be used, The incidence of growth arrest following physeal ankle injuries is high and long- term follow-up is indicated, REFERENCES: Cass JR, Peterson HA: Salter-Harris type-IV injuries of the distal tibial epiphyseal growth plate, with emphasis on those involving the medial malleolus. J Bone Joint Surg Am 1983;65:1059-1070. Barmada A, Gaynor T, Mubarak SJ: Premature physeal closure following distal tibia physcal fractures: A new radiographic predictor. J Pediatr Orthop 2003;23:733-739, 2010 Pediatric Orthopaedic Examination Answer Book + 61 Question 69 What is the most likely reason open fractures tend to heal more slowly than closed fractures? 1Loss of osteoinductive potential from the hematoma that is lost around the fracture Introduction of foreign material ‘Subclinical infection Loss of blood supply at the fracture site Loss of soft-tissue coverage at the fracture site PRN PREFERRED RESPONSE: 1 DISCUSSION: In open fractures, the hematoma that forms beneath the periosteum and around the ends of the fracture site is lost from the open wound, In addition, the irrigation process washes out the hematoma that contains growth factors and cytokines from the platelets. While loss of blood supply at the fracture site and soft-tissue coverage are important factors, the most important is loss of the factors that initiate the inflammatory phase of fracture healing. Infection may also delay healing, but is less common in this population. REFERENCES: Buckwalter JA, Einhorn TA, Simon SR (eds): Orthopaedic Basic Science: Biology and Biomechanics of the Musculoskeletal System, ed 2. Rosemont, IL, American Academy of Orthopaedic Surgeons, 2000, pp 377-381. Green NE, Swiontkowski MF (eds): Skeletal Trauma in Children, ed 3. Philadelphia, PA, WB Saunders, 2003, pp 1-14. Question 70 ‘A patient with Pott’s discase, tuberculosis of the spine, is more likely to have which of the following early findings? ‘Acute onset back pain and neurologic dysfunction Preservation of the disk space between two affected adjacent end plates Involvement of the cervical spine and torticallis Elevated WBC count and markedly elevated erythrocyte sedimentation rate Lordotic deformity in late stages of the disease paepe PREFERRED RESPONSE: 2 DISCUSSION: Tuberculosis of the spine typically has an indolent presentation. Unlike pyogenic infections of the spine, the disk space is usually preserved, Most commonly, the thoracic and lumbar spine are affected. Laboratory studies may be nonspecific, Delayed presentation usually results in neurologic compromise and a kyphotic deformity, Treatment includes a multidrug regimen. Surgery is indicated for deformity correction or failure of medical treatment. REFERENCES: Rajasekaran S: Buckling collapse of the spine in childhood spinal tuberculosis. Clin Orthop Relat Res 2007;460:86-92. Tay BK, Deckey J, Hu SS: Spinal infections. J Am Acad Orthop Surg 2002;10:188-197. (62+ American Academy of Orthopaedic Surgeons Figure 71a Figure 71b Figure 71c Figure 71d Question 71 A.10-year-old child was referred for spinal curvature and a 2-year history of back pain. She has pain during the day and pain at night that wakes her from sleep and is temporarily relieved with nonsteroidal anti-inflammatory drugs. Examination shows very tight hamstrings and an irritative spinal curvature. Figures 71a through 71d show radiographs, a bone scan, and a CT scan, What is the most appropriate treatment? \. Bracing with a thoracolumbosacral orthosis (TLSO) 2, Observation with repeat radiographs of the scoliosis in 3 months and nonsteroidal anti- inflammatory drugs for the pain 3. MRI of the neuro-axis 4, Surgical removal 5. Radiofrequency ablation PREFERRED RESPONSE: 4 DISCUSSION: The history, examination findings, and studies are consistent with an osteoid osteoma. The CT scan shows a classic “target” lesion, and the bone scan has intense uptake at the site of the osteoid osteoma. The child has had a 2-year history of pain that even wakes her from sleep, so observation and anti-inflammatory drugs is not a preferred treatment. Bracing will not help with the discomfort because the pain is not mechanical in nature. MRI would not be needed in addition to the studies already completed. The osteoid osteoma is close to the spinal cord so radiofrequency ablation is not preferred. Surgical removal and biopsy is the treatment of choice. REFERENCES: Frassica FJ, Waltrip RL, Sponseller PD, ct al: Clinicopathologic features and treatment of ‘osteoid osteoma and osteoblastoma in children and adolescents. Orthop Clin North Am 1996;27:559-574. Cantwell CP, Obyme J, Eustace $: Current trends in treatment of osteoid osteoma with an emphasis on radiofrequency ablation. Eur Radiol 2004;14:607-617. 2010 Pediatrie Orthopaedic Examination Answer Book + 63 Figure 72 Question 72 ‘A.10%-year-old boy sustained the injury shown in Figure 72 when he fell out of a tree. This is a closed, neurologically intact injury and the patient has no head injury or loss of consciousness. He weighs 115 pounds and is otherwise healthy. What is the optimal treatment option for this injury? Immediate spica casting Flexible intramedullary nail placement Traction and casting Extemnal fixation Solid intramedullary nail fixation via the greater trochanter pyrene PREFERRED RESPONSE: 5 DISCUSSION: Although flexible intramedullary nails are a good treatment alternative for femoral shaft fractures in older children, patients weighing more than 100 pounds have a higher incidence of complications that include bending of the nails. Therefore, transtrochanteric solid intramedullary nail fixation is most likely the best option for this patient. Using a greater trochanteric entry point avoids the piriformis fossa and the possibility of osteonecrosis. External fixation is not a good altemnative for this, patient because of the transverse nature of the fracture, External fixation of this fracture pattern has been associated with a high refracture rate, Traction and casting can be performed but results in a lengthy hospital stay and a very large cast in an overweight 10-year-old child. REFERENCES: Flynn JM, Schwend RM: Management of pediatric femoral shaft fractures. J Am Acad Orthop Surg 2004;12:347-359. Gordon JE, Swenning TA, Burd TA, et al: Proximal femoral radiographic changes after lateral transtrochanteric intramedullary nail placement in children. J Bone Joint Surg Am 2003;85:1295-1301. 62+ American Academy of Orthopaedic Surgeons Figure 71a Figure 71b Figure Tle Figure 71d Question 71 A 10-year-old child was referred for spinal curvature and a 2-year history of back pain. She has pain during the day and pain at night that wakes her from sleep and is temporarily relieved with nonsteroidal anti-inflammatory drugs. Examination shows very tight hamstrings and an irritative spinal curvature. Figures 71a through 71d show radiographs, a bone scan, and a CT scan, What is the most appropriate treatment? Bracing with a thoracolumbosacral orthosis (TLSO) Observation with repeat radiographs of the scoliosis in 3 months and nonsteroidal anti- inflammatory drugs for the pain 3. MRI of the neuro-axis 4. Surgical removal 5. Radiofrequency ablation Re PREFERRED RESPONSE: 4 DISCUSSION: The history, examination findings, and studies are consistent with an osteoid osteoma. The CT scan shows a classic “target” lesion, and the bone scan has intense uptake at the site of the osteoid osieoma. The child has had a 2-year history of pain that even wakes her from sleep, so observation and anti-inflammatory drugs is not a preferred treatment, Bracing will not help with the discomfort because the pain is not mechanical in nature. MRI would not be needed in addition to the studies already completed. The osteoid osteoma is close to the spinal cord so radiofrequency ablation is not preferred. Surgical removal and biopsy is the treatment of choice. REFERENCES: Frassica FJ, Waltrip RL, Sponseller PD, et al: Clinicopathologic features and treatment of ‘osteoid osteoma and osteoblastoma in children and adolescents. Orthop Clin North Am 1996;27:559-574, Cantwell CP, Obyme J, Eustace S: Current trends in treatment of osteoid osteoma with an emphasis on radiofrequency ablation. Eur Radiol 2004;14:607-617. 2010 Pediatric Orthopaedic Examination Answer Book + 63 Question 72 A 10'%4-year-old boy sustained the injury shown in Figure 72 when he fell out of a tree. This is a closed, neurologically intact injury and the patient has no head injury o loss of consciousness. He weighs 115 pounds and is otherwise healthy. What is the optimal treatment option for this injury? Immediate spica casting Flexible intramedullary nail placement Traction and casting External fixation Solid intramedullary nail fixation via the greater trochanter paero PREFERRED RESPONSE: 5 DISCUSSION: Although flexible intramedullary nails are a good treatment alternative for femoral shaft fractures in older children, patients weighing more than 100 pounds have a higher incidence of complications that include bending of the nails. Therefore, transtrochanteric solid intramedullary nail fixation is most likely the best option for this patient. Using a greater trochanteric entry point avoids the piriformis fossa and the possibility of osteonecrosis. External fixation is not a good alternative for this patient because of the transverse nature of the fracture, External fixation of this fracture pattern has been associated with a high refracture rate. Traction and casting can be performed but results in a lengthy hospital stay and a very large cast in an overweight 10-year-old child. REFERENCES: Flynn JM, Schwend RM: Management of pediatric femoral shaft fractures. J Am Acad Orthop Surg 2004;12:347-359, Gordon JE, Swenning TA, Burd TA, et al: Proximal femoral radiographic changes after lateral transtrochanteric intramedullary nail placement in children. J Bone Joint Surg Am 2003;85:1295-1301. 64+ American Academy of Orthopaedic Surgeons Figure 73a Figure 73b Question 73 A 15-year-old right-handed pitcher reports shoulder pain after throwing. His symptoms have been present for 3 months and have been getting progressively worse. Clinical examination shows no atrophy of the shoulder muscles, but he has pain with resisted motion of the shoulder, especially intemal rotation, Radiographs are shown in Figures 73a and 73b. What is the next step in the evaluation and treatment of his shoulder pain? ‘MRVWarthrogram of the right shoulder CT of the right proximal humerus Bone biopsy of the right proximal humerus Cessation of throwing for 6 to 8 weeks, followed by a progressive throwing program Arthroscopic evaluation of the right shoulder yeep PREFERRED RESPONSE: 4 DISCUSSION: The patient has proximal humeral epiphyseolysis, otherwise known as “Little League shoulder.” This is an overuse injury of the shoulder in the skeletally immature overhead throwing athlete. Most frequently seen in pitchers, it usually develops after an increase in the amount or intensity of throwing activity. Initial treatment involves cessation of throwing activities so the proximal humeral growth plate injury can heal, followed by a gradual return to throwing, REFERENCES: Chen FS, Diaz VA, Loebenberg M, et al: Shoulder and elbow injuries in the skeletally immature athlete. J Am Acad Orthop Surg 2005;13:172-185, Keeley DW, Hackett T, Keimns M, et al: A biomechanical analysis of youth pitching mechanics. J Pediatr Orthop 2008;28:452-459, Sabick MB, Kim YK, Torry MR, et al: Biomechanics of the shoulder in youth baseball pitchers: Implications for the development of proximal humeral epiphysiolysis and humeral retrotorsion. Am J Sports Med 2005;33:1716-1722. 2010 Pediatric Orthopaedic Examination Answer Book + 65 Question 74 A 12-year-old child with Duchenne’s muscular dystrophy has a 40-degree scoliotic deformity. Prior to surgery, the orthopaedic surgeon should wait for further progression. request a hematology consult request a neurology consult, request a cardiology consult. implement a 6-month trial of bracing. yeenm PREFERRED RESPONSE: 4 DISCUSSION: In Duchenne’s muscular dystrophy, spinal deformities are common. Spinal deformity usually develops as a child begins sitting in the preteen years. Unlike adolescent idiopathic scoliosis, scoliosis in Duchenne’s muscular dystrophy is treated early; spinal fusion for a 40-degree deformity is not unusual, Although hematology and neurology consults usually are not necessary prior to surgery, every child should have a comprehensive cardiac evaluation, including an EKG and an echocardiogram because cardiomyopathy is part of the pathologic spectrum of Duchenne’s muscular dystrophy requiring preoperative assessment and intervention. REFERENCE: Fischgrund JS (ed): Orthopedic Knowledge Update 9. Rosemont, {L, American Academy of Orthopaedic Surgeons, 2008, p 790. Question 75 An 8-month-old child is seen in the emergency department with seizures and a fractured femur. The ‘mother states that the child fell off the bed at the babysitter’s house, There are bilateral bruises on the anterior and posterior chest walls. Retinal hemorthages are present, The temperature is 98.9 degrees F (37.2 degrees C). What is the most likely diagnosis? Febrile seizure Fractured skull Subdural hematoma Shaken baby syndrome Contracoup brain injury ween PREFERRED RESPONSE: 4 DISCUSSION: Shaken baby syndrome is associated with chest ecchymosis and head trauma. Retinal hemorthages are often found, but are not pathognomonic. Contracoup injury was originally implicated, but more recent evidence shows that the head is actually struck against a hard object that causes a subdural hematoma, 6 + American Academy of Orthopaedic Surgeons REFERENCES: LeFanu J, Edwards-Brown R: Patterns of presentation of shaken baby syndrome: Subdural and retinal haemorrhages are not necessarily signs of abuse. BMJ 2004;328:767. Richards PG, Bertocci GE, Bonshek RE, et al: Shaken baby syndrome, Arch Dis Child 2006;91:205-206. Question 76 A 12-year-old boy with a family history of neurofibromatosis has anterolateral bowing of the left tibia. He has no pain and is ambulatory. Radiographs show a narrowed medullary canal but intact cortices. Treatment should consist of which of the following? Ankle-foot orthosis with anterior shell Vascularized fibular graft Intramedullary nailing of the left tibia Amputation Physical therapy PREFERRED RESPONSE: | veer DISCUSSION: Anterolateral bowing of the tibia is associated with confirmed neurofibromatosis in approximately 50% of patients. Although the risk of fracture with the development of pseudarthrosis exists, the initial treatment consists of bracing through maturity. REFERENCES: Vander Have KL, Hensinger RN, Caird M, et al: Congenital pseudarthrosis of the tibia. J Am Acad Orthop Surg 2008;16:228-236, Vitale MG, Guha A, Skaggs DL: Orthopaedic manifestations of neurofibromatosis in children: An update. Clin Orthop Relat Res 2002;401:107-118. Question 77 ‘A4-year-old boy has had an isolated painful limp for the past month, He is diagnosed with Lege-Calve- Perthes disease (CPD) that involves nearly all of his capital femoral epiphysis. Which of the following best describes his prognosis? ‘A spherical, painless hip at maturity An incongruous hip joint at maturity Likely spontaneous hip fasion A30% to 40% chance of a poor outcome Rapid recovery with minimal sequelae veer PREFERRED RESPONSE: 4 DISCUSSION: This young child with total head involvement LCPD is at some risk of a poor outcome due to the extent of his disease. Most children of this age will recover well with a good outcome. He is more likely to end up with a spherical femoral head than an older child with the same extent of involvement, 2010 Pediatric Orthopaedic Examination Answer Book + 67 REFERENCES: Schoenecker PL, Stone JW, Capelli AM: Legg-Perthes disease in children under 6 years ‘old, Orthop Rev 1993;22:201-208. Rosenfeld SB, Herring JA, Chao JC: Legg-Calve-Perthes disease: A review of cases with onset before six years of age. J Bone Joint Surg Am 2007;89:2712-2722. Taal Figure 78 Question 78 Figure 78 shows the radiograph of a 4-year-old girl who has progres include which of the following? bow legs. Management should 1. Bracing 2. Observation 3. Tibial osteotomy 4. Physeal bridge resection 5. Lateral physeal hemiepiphysiodesis plate PREFERRED RESPONSE: 3 DISCUSSION: A diagnosis of Blount’s disease is indicated by the abnormal shape of the medial metaphysis of the tibia, the progressive nature of the deformity, and the focal nature of the angulation. A 4-year-old child with Blount’s disease should undergo surgical correction consisting of a tibial osteotomy before there is a permanent growth arrest that would require physeal bridge resection and/or repeated ‘osteotomies. With a Langenskiold type IV lesion, bracing or hemiepiphyseal plate fixation is not expected to correct the deformity. REFERENCES: Schoenecker PL, Meade WC, Pierron RL, et al: Blount’s disease: A retrospective review and recommendations for treatment. J Pediatr Orthop 1985;2:181-186. Langenskiold A: Tibia Vara: A critical review. Clin Orthop Relat Res 1989;246:195-207. Bowen RE, Dorey FJ, Moseley CF: Relative tibial and femoral varus as a predictor of varus deformities of the lower limbs in young children. J Pediatr Orthop 2002;22:105-111. 68 + American Academy of Orthopaedic Surgeons Figure 79 Question 79 A 12-year-old girl has the painful foot deformity seen in Figure 79. You advise her that she has juvenile bunions. How do they differ from adult bunions? ‘Metatarsus primus varus Large exostosis Rigidity of the metatarsal phalangeal joint Greater hallux valgus angle than in adult bunions Prominent bursal thickening over the medial eminence yeepe PREFERRED RESPONSE: | DISCUSSION: The hallmark of the juvenile bunion is metatarsus primus varus. Increased flexibility of the first metatarsal phalangcal joint leads to increased deformity, The hallux valgus angle is less than the adult bunion, Bursal thickenings and prominence of the medial eminence are less in a juvenile bunion. REFERENCES: Coughlin MJ: Juvenile bunions, in Mann RA, Coughlin MJ (eds): Surgery of the Foot and Ankle, ed 6. Philadelphia, PA, Mosley, 1993, pp 297-339. ‘Coughlin MJ, Mann RA: The pathophysiology of juvenile bunion. Instr Course Lect 1987;36:123-136. 2010 Pediatric Orthopaedic Examination Answer Book + 69 wee Figure 80a Figure 80b Figure 80c Question 80 A 12-year-old girl is seen after tripping and twisting her ankle earlier in the morning. She had immediate pain and swelling and was unable to bear weight. Radiographs are shown in Figures 80a through 80c. Appropriate treatment should consist of which of the following? Short leg cast with no weight bearing for the first 3 weeks Short leg cast with immediate weight bearing Long leg cast with no weight bearing for the first 3 weeks Open reduction and internal fixation, avoiding the physeal growth plate and joint CAM walker with immediate weight bearing wpene PREFERRED RESPONSE: 4 DISCUSSION: Salter Harris III and IV fractures of the medial malleolus often contain a large cartilaginous portion, larger than the apparent ossified fragment seen on radiographs. Consequently, articular incongruity is common. As such, open reduction and internal fixation is often required. Growth disturbance and angular deformity are also common complications of Salter Harris III and IV fractures of the distal tibia, REFERENCES; Flynn JM, Skaggs DL, Sponseller PD, et al: The surgical management of pediatric fractures of the lower extremity. Instr Course Lect 2003;52:647-659. Kay RM, Matthys GA: Pediatric ankle fractures: Evaluation and treatment. J Am Acad Orthop Surg 2001;9:268-278, 70 + American Academy of Orthopaedic Surgeons Question 81 A 6-year-old child is seen in the emergency department after falling from the monkey bars. Examination reveals tenderness of the right humerus and an inability to dorsiflex the wrist. No other injuries are identified. Radiographs show a minimally displaced and angulated (10 degrees of varus angulation) fracture of the distal one third of the humeral shaft. Initial management should consist of which of the following? Immediate exploration of the radial nerve and cast application Immediate exploration of the radial nerve, with percutaneous Kirschner wire fixation Immediate exploration of the radial nerve with open reduction and plate fixation ‘Monitoring of radial nerve function and application of a sling and swathe Monitoring of radial nerve function and external fixation, veers PREFERRED RESPONSE: 4 DISCUSSION: Humeral shaft fractures in children rarely require open reduction, Shoulder and elbow function does not appear to be affected by up to 40 degrees of angulation in this patient population. Because of the high rate of remodeling in pediatric patients, the standard treatment is immobilization ina sling and swatbe, a hanging arm cast, or a compressive dressing. Surgical fixation of humeral shaft fractures is usually only necessary in open injuries, multitrauma, or severely displaced fractures. Most radial nerve injuries associated with humerus fractures are secondary to contusion. Almost all associated radial nerve injuries in pediatric patients can be treated with observation. REFERENCES: Abel MF (ed): Orthopaedic Knowledge Update: Pediatrics 3. Rosemont, IL, American Academy of Orthopaedic Surgeons, 2006, p 304. Shrader MW: Proximal humerus and humeral shaft fractures in children. Hand Clin 2007;23:431-435. Caviglia H, Garrido CP, Palazzi FF, et al: Pediatric fractures of the humerus. Clin Orthop Relat Res 2005;432:49-56. Question 82 In infantile idiopathic scoliosis, which of the following factors suggests progression? Age at presentation Rib overlap of the apical vertebra Rib vertebral angle difference of greater then 10 degrees Male gender Family history vaepe PREFERRED RESPONSE: 2 2010 Pediatric Orthopaedic Examination Answer Book + 71 DISCUSSION: Infantile idiopathic scoliosis occurs more commonly in boys, with a 3 to 1 male to female ratio, Neural axis abnormalities, hip dysplasia, and congenital heart disease are all associated with the condition; spontaneous correction frequently occurs. Curve progression can be predicted by the rib vertebral angle difference or the phase of the rib head, Rib overlap of the apical vertebral body or a rib vertebral angle difference of greater than 20 degrees indicates that the curve is likely to progress. Gender, family history, and age at presentation have not been found to be risk factors for progression. REFERENCES: Mehta MH: The rib-vertebra angle in the early diagnosis between resolving and progressive infantile scotiosis. J Bone Joint Surg Br 1972:54:230-243, Fischgrund JS (ed): Orthopedic Knowledge Update 9. Rosemont, IL, American Academy of Orthopaedic Surgeons, 2008, p 697. Question $3 ‘A3-year-old child sustains a 12/T3 fracture-dislocation with complete paraplegia secondary to a car accident in which the child was an unrestrained passenger. What is the likelihood that this child will develop subsequent spinal deformity in the future? 0% if bracing is used 25% 50% 75% 90% or greater peeN PREFERRED RESPONSE: 5 DISCUSSION: More than than 90% of preadotescent children who sustain a significant spinal cord injury subsequently develop scoliosis. Conversely, progressive paralytic spinal deformity is uncommon in the postadolescent patient. Bracing has not been shown to be effective in the prevention of scoliosis in the preadolescent patient with spinal cord injury. REFERENCES: Mayfield JK, Erkkila JC, Winter RB: Spine deformity subsequent to acquired childhood spinal cord injury. J Bone Joint Surg Am 1981;63:1401-1411 Dearolf WW ILL, Betz RR, Vogl LC, et al: Scoliosis in pediatric spinal cord-injured patients, J Pediatr Orthop 1990;10:214-218. Mehta S, Betz RR, Muleahey MJ, et al: Effect of bracing on paralytic scoliosis secondary to spinal cord injury. J Spinal Cord Med 2004;27:S88-S92 72+ American Academy of Orthopaedic Surgeons Question 84 ‘What is the most common causative bacteria in septic arthritis in children? Staphylococcus aureus Brucella melitensis Haemophilus influenzae Kingella kingae Streptococcus pneumonia PREFERRED RESPONSE: 1 yee DISCUSSION: The spectrum of causative bacteria and frequency of occurrence of specific pathogens in septic arthritis are similar to those seen in osteomyelitis, with Staphylococcus aureus being the most common, Other common causative organisms include Kingella Kingae, Streptococcus pneumonia, Klebsiella species, Salmonella, Brucella melitensis, and Haemophilus influenzae. REFERENCES: Herring JA: Tachdjian’s Pediatric Orthopaedics, ed 4. Philadelphia, PA, WB Saunders, 2008, p 2109. Jackson MA, Nelson JD: Etiology and m in pediatric patients. J Pediatr Orthop 1982;2:. | management of acute suppurative bone and joint infections 3-323. 2010 Pediatrie Orthopaedic Examination Answer Book * 73, Figure 85 ‘Question 85, A 10-year-old girl fell from her bike and now reports pain and swelling in the left knee and pain with weight bearing. Examination reveals a left knee effusion and pain with range of motion. A radiograph is shown in Figure 85, Treatment should consist of a long leg cast in extension. a long leg cast in 10 degrees of flexion. closed reduction and long leg casting in 10 degrees of flexion aspiration of the hemarthrosis for comfort and a knee immobilizer for 6 weeks open or arthroscopic reduction and internal fixation yaeyr PREFERRED RESPONSE: 5 DISCUSSION: The child has a type III tibial spine avulsion fracture. When the avulsed fragment is completely displaced, the preferred treatment is open or arthroscopic reduction of the fragment and internal fixation with sutures or screws, Type I fractures are nondisplaced and can be treated with a long, leg cast; type IT fractures are hinged and can be treated in a long leg cast if closed reduction is successful. Many patients have some objective anterior cruciate ligament laxity after a tibial spine avulsion fracture; however, with adequate treatment most patients do not have symptomatic laxity. REFERENCES: Mah JY, Adili A, Otsuka NY, et al: Follow-up study of arthroscopic reduction and fixation of type II] tibial-eminence fractures. J Pediatr Orthop 1998;18:475-477. McLennen JG: Lessons leamed after second-look arthroscopy in type III fractures of the tibial spine. J Pediatr Orthop 1995;15:59-62. Meyers MH, McKeever FM: Fracture of the intercondylar eminence of the tibia. J Bone Joint Surg Am 1970;52:1677-1684. 74+ American Academy of Orthopaedic Surgeons Question 86 Anon-communicative 16-year-old girl with spastic quadriplegic cerebral palsy and a 75-degree thoracolumbar scoliosis undergoes a successful posterior spinal fusion with instrumentation. What is the most predictable outcome of the surgical procedure? Improved cognitive function Improved caregiver satisfaction Improved nutrition Decreased pain Improved mobility yaepe PREFERRED RESPONSE: 2 DISCUSSION: Surgical treatment of spinal deformity in a totally involved child with cerebral palsy has been shown on outcomes instruments to significantly improve the caregiver’s perception of the child’s comfort. The other parameters mentioned are difficult to measure and unpredictable. REFERENCES: Tsirikos AI, Lipton G, Chang WN, et al: Surgical correction of scoliosis in pediatric patients with cerebral palsy using the unit rod instrumentation. Spine 2008;33:1133-1140. Cassidy C, Craig CL, Perry A, et al: A reassessment of spinal stabilization in severe cerebral palsy. J Pediatr Orthop 1994;14:731-739, Figure 87 Question 87 A 12-year-old boy has had left thigh pain for the past 4 months. Examination shows lack of internal rotation and abduction, and external rotation with hip flexion. A radiograph is shown in Figure 87. What is the most appropriate treatment? Physical therapy In situ pinning Reduction and percutaneous pinning Surgical dislocation of the hip with reduction under direct vision Spica casting PREP E PREFERRED RESPONSE: 2 2010 Pediatric Orthopaedic Examination Answer Book + 75 DISCUSSION: The patient has a stable slipped capital femoral epiphysis (SCFE). Preferred treatment of stable SCFE is in situ pinning. In situ fixation of stable SCFE has an extremely low rate of osteonecrosis. Gentle postural reduction with hip capsulotomy or surgical dislocation of the hip with reduction has been advocated for unstable SCFE. REFERENCES: Aronson DD, Peterson DA, Miller DV: Slipped capital femoral epiphysis: The case for internal fixation in situ. Clin Orthop Relat Res 1992;281:115-122, Loder RT, Richards BS, Shapiro PS, et al: Acute slipped capital femoral epiphysis: The importance of physeal stability. J Bone Joint Surg Am 1993;75:1134-1140. Figure 88a Figure 88 Question 88 A4-year-old girl falls off swing and injures her right elbow. The radiographs are shown in Figures 88a and 88b. What is the most likely diagnosis? 1. Displaced right olecranon fracture 2. Right elbow dislocation with spontaneous reduction and entrapped medial epicondyle fragment 3. Effusion of the right elbow without an identifiable radiographic fracture 4. Nondisplaced right lateral condyle fracture of the distal humerus 5. Right Gartland type I supracondylar humerus fracture PREFERRED RESPONSE: | DISCUSSION: To accurately diagnose skeletal injuries around the elbow in children, the practitioner must be knowledgeable about the progressive ossification centers, In this case, at age 4 years, a female will have a partially ossified capitellum, radial head, and medial epicondyle, The trochlea, olecranon, and lateral epicondyle ossification centers should remain fully cartilaginous at this stage of development. The irregularity in the area of the olecranon on the radiograph represents a displaced fracture requiring accurate reduction and fixation to ensure restoration of articular congruity and full elbow function. The presence of a posterior fat pad sign on the radiograph is indicative of a traumatic effusion, but there is an identifiable fracture in this patient. Olecranon fractures are commonly seen in children with osteogenesis, imperfecta. 76 + American Academy of Orthopaedic Surgeons REFERENCES: Cheng JC, Wing-Man K, Shen WY, et al: A new look at the sequential development of elbow-ossification centers in children. J Pediatr Orthop 1998;18:161-167. Parent S, Wedemeyer M, Mahar AT, et al: Displaced olecranon fractures in children: A biomechanical analysis of fixation methods. J Pediatr Orthop 2008;28:147-151. Beaty JH, Kasser JR (eds): Rockwood and Green’s Fractures in Children, ed 6. Philadelphia, PA, Lippincott Williams and Wilkins, 2006, pp 591-660. Question 89 ‘A 5-year-old boy is seen in the emergency department with a 2-day history of refusing to walk. Examination shows that he as a temperature of 102.2 degrees F (39 degrees C) and limited range of ‘motion of the right hip. The AP pelvic radiograph is normal, The WBC count is normal but the C-reactive protein and erythrocyte sedimentation rate (ESR) are elevated. What is the next step in management? IV antibiotics Oral antibioties Ibuprofen Observation and repeat evaluation in 2 weeks Aspiration of the right hip yaeee PREFERRED RESPONSE: 5 DISCUSSION: The history, physical examination, and laboratory studies suggest a septic hip. Recent studies indicate that a child with elevated ESR, a WBC count of greater than 12,000/mm3, a temperature of greater than 38.5 degrees, and unwillingness to walk is very likely to have septic arthritis of the hip versus toxic synovitis. The best way to confirm the diagnosis is by hip aspiration. No medications should be started until a diagnosis is made. Toxic synovitis is common, but significantly less likely if three of the above criteria are present, This condition usually responds well to ibuprofen, but requires close observation, Septic hips are considered urgent conditions and therefore a repeat evaluation in 2 weeks is inappropriate. REFERENCES: Herring JA: Tachdjian’s Pediatric Orthopaedics, ed 4. Philadelphia, PA, WB Saunders, 2008, pp 2109-2113. ‘Abel MF (ed): Orthopaedic Knowlede Update: Pediatrics 3. Rosemont, IL, American Academy of Orthopaedic Surgeons, 2006, pp 62-65. Kocher MS, Mandiga R, Murphy JM, et al: A clinical practice guideline for treatment of septic arthritis in children: Efficacy in improving process of care and effect on outcome of septic arthritis of the hip. J Bone Joint Surg Am 2003;85:994-999, Kocher MS, Mandiga R, Zurakowski D, et al: Validation of a clinical prediction rule for the differentiation between septic arthritis and transient synovitis of the hip in children. J Bone Joint Surg Am 2004;86:1629-1635. 2010 Pedliatric Orthopaedic Examination Answer Book + 77 Question 90 Abnormal autosomal-dominant genes typically result in what type of defect? Structural Enzymatic Biochemical ‘Translocations Deletions peene PREFERRED RESPONSE: | DISCUSSION: Autosomal-dominant gene defects usually cause structural deformities. An example is achondroplasia where the dominant gene codes for fibroblast growth factor receptor 3, a structural protein that results in the quantitative decrease in cartilage formation. The gene defects are inherited from one of the parents or are sporadic mutations. Autosomal-recessive genes code for enzymatic and biochemical defects, Translocations and deletions relate to chromosomal abnormalities. REFERENCE: Einhorn TA, O’Keefe RJ, Buckwalter JA (eds): Orthopaedic Basic Science: Foundations of Clinical Practice, ed 3. Rosemont, IL, American Academy of Orthopaedic Surgeons, 2007, pp 86-99. Question 91 ‘A.10-year-old boy is struck by a car and sustains open left tibia and fibula fractures with bone protruding through a 7-cm laceration, multiple deep and superficial abrasions over the anterior leg, and road gravel is, present in the wounds. His foot is warm and well-perfused with normal sensation and he has no pain with passive range of motion of the toes. Optimal treatment should consist of irrigation and debridement of the fractures and application of an external fixator. irrigation and debridement of the fractures and a reamed intramedullary nail. irrigation and debridement of the fracture and percutaneous Kirschner wire fixation. submuscular plating, reduction and a short leg cast. PREFERRED RESPONSE: | yay DISCUSSION: The patient has a grade 2 open fracture and therefore needs wound debridement as a first step, followed by fracture stabilization preferably with an external fixator. A reamed intramedullary nail is not indicated in a 10-year-old child with open growth plates. Submuscular plating is not needed in an open fracture and there is no mention of fracture debridement, Percutaneous Kirschner wires will not provide adequate fracture stabilization, nor will a short leg cast, Flexible nailing should be considered as another form of fixation. REFERENCES: Buckley SL, Smith G, Sponseller PD, et al: Open fractures of the tibia in children. J Bone Joint Surg Am 1990;72:1462-1469. Song KM, Sangeorzan B, Benirschke S, et al: Open fractures of the tibia in children. J Pediatr Orthop 5-639. 78 + American Academy of Orthopaedic Surgeons Question 92 Brace treatment is recommended for adolescent idiopathic scoliosis when which of the following findings is present? Any patient with a curve of greater than 25 degrees Boys with a curve of greater than 20 degrees Premenarchal girls with a curve of greater than 30 degrees More than 5 degrees of progression in a growing child with a 20-degree curve A girl who is Risser 4 with a 30-degree curve wayne PREFERRED RESPONSE: 3 DISCUSSION: Brace treatment is recommended for a patient with substantial growth potential (Risser 2), any curve of greater than 30 degrees, or for a patient with a curve of greater than 20 to 25 degrees with more than 5 degrees of documented progression, Skeletally immature patients who have a curve of greater than 25 degrees are not considered for bracing, If'a boy has a curve of 20 degrees, progression would be necessary to recommend bracing. A premenarchal girl with the curve of greater than 45 degrees is best treated with early surgical intervention, as progression is extremely likely. A girl who is Risser 4 with a 30-degree curve on presentation should be observed. REFERENCE: Fischgrund JS (ed): Orthopedic Knowledge Update 9. Rosemont, IL, American Acaderny of Orthopaedic Surgeons, 2008, p 698. 2010 Pediatric Orthopaedic Examination Answer Book * 79 Figure 93a Figure 93b Question 93 ‘A 75-Ib, 10-year-old boy fell and is now unable to walk and has left thigh pain and deformity, He has no other injuries and is otherwise healthy. Radiographs are shown in Figures 93a and 93b, What is the most appropriate management? Retrograde elastic nail fixation Traction now and a hip spica cast in 7 to 10 days Antegrade intramedullary rod fixation External fixation Open reduction and plate fixation veene PREFERRED RESPONSE: | DISCUSSION: The fracture is best treated with elastic nails. Traction and a spica cast could be used but requires more time in the hospital and the cast increases family inconvenience for 4 to 6 weeks, There are lateral entry nails that could be used, but to date there is still not a long enough follow-up to determine the risk of osteonecrosis, External fixation may be an altemative for a patient with multiple injuries or extensive soft-tissue damage. Open plating could also be used but in the midshaft location, flexible nails are the most appropriate choice and offer the fewest number of potential complications. REFERENCES: Abel MF (ed): Orthopaedic Knowledge Update: Pediatrics 3. Rosemont, IL, American Academy of Orthopaedic Surgeons, 2006, pp 271-279. Buechsenschuetz KE, Mehlman CT, Shaw KJ, et al: Femoral shaft fractures in children: Traction and casting versus elastic stable intramedullary nailing. J Trauma 2002;53:914-921 80 + American Academy of Orthopaedic Surgeons Figure 94 Question 94 ‘A 7-year-old boy is seen for follow-up for a scoliotic deformity. His parents are concemed because his deformity seems to have increased. He has no pain and is neurologically intact. A radiograph is shown. in Figure 94, and measurement of his curve reveals that it has increased 10 degrees. What is the most appropriate recommendation for this patient at this time? Observation Bracing A“growing rod” Distraction instrumentation and posterior arthrodesis Hemivertebra excision and limited fusion yaeen PREFERRED RESPONSE: 5 DISCUSSION; Nakamura and associates have reported good results in patients with resection for hemivertebra-related congenital scoliosis who have a progression of their deformity. Because of the progression, observation is not appropriate for this patient’s deformity. Bracing has not been shown to alter the progression of congenital scoliosis. The “growing rod” technique is also not effective in preventing progression related to hemivertebra. Distraction instrumentation carries an increased risk of neurologic complications in children with congenital spine deformities. Progression after posterior arthrodesis alone can occur through the so-called “crankshaft phenomenon.” REFERENCES: Nakamura H, Matsuda H, Konishi S, et al: Single-stage excision of hemivertebrae via the posterior approach alone for congenital spine deformity: Follow-up period longer than ten years. Spine 2002;27:110-115. RufM, Harms J: Posterior hemivertebra resection with transpedicular instrumentation: Early correction in children aged 1 to 6 years. Spine 2003;15:2132-2138. 2010 Pediatric Orthopaedic Examination Answer Book + 81 Figure 95a Figure 95b Figure 95c Figure 95d Question 95 A 12-year-old boy is seen 1 week after injuring his knee while playing soccer. He notes pain and swelling. Examination reveals an effusion, laxity with Lachman testing, and he walks with a limp. Radiographs and an MRI scan are shown in Figures 95a through 95d. Treatment should consist of which of the following? 1. Physical therapy 2. Anterior cruciate ligament reconstruction 3. Anterior cruciate ligament reconstruction when the child reaches skeletal maturity 4. Anatomic reduction and internal fixation of the fracture fragments 5. Casting in extension PREFERRED RESPONSE: 4 DISCUSSION: The radiographs and MRI scan show a displaced tibial eminence fracture. Meyer and McKeever classified these injuries, with type | being a nondisplaced tibial eminence fracture; type 2 being a displaced tibial eminence fracture with a posterior hinge, and type 3 being a displaced tibial eminence fracture, Tibial eminence fractures in children are equivalent to anterior cruciate ligament tears in adults. Treatment should be anatomic reduction, which often requires an arthroscopic or open procedure, followed by fixation. REFERENCES: Green NE, Swiontkowski MF: Skeletal Trauma in Children, ed 3. Philadelphia, PA, WB Saunders, 2003, pp xvi, 452-455, 638. Zionts LE: Fractures around the knee in children, J Am Acad Orthop Surg 2002;10:345-355. 82+ American Academy of Orthopaedic Surgeons Figure 96 Question 96 ‘A teenager had pain in the left buttock while running the hurdles. He was treated with 4 weeks of rest and crutch walking, and then started physical therapy for stretching and muscle strengthening, Nine months later he now reports pain with sitting and has not been able to resume running or sports activity. Figure 96 shows a radiograph of the pelvis. Treatment should consist of which of the following? Continued physical therapy Spica casting for 6 weeks Biopsy of the lesion Steroid injection into the area of discomfort Excision of the fragment PRY PE: PREFERRED RESPONSE: 5 DISCUSSION: The patient has an established nonunion of the ischial tuberosity. Avulsion fractures of the pelvis are generally treated with rest and symptomatic treatment, Avulsion fractures of the ischial tuberosity are the most prone to nonunion. Most patients have few symptoms but some have trouble sitting and returning to sports. Excision of the avulsed fragment or open reduction and internal fixation are indicated for painful nonunions of the ischial tuberosity. REFERENCES: Fembach SK, Wilkinson RH: Avulsion injuries of the pelvis and proximal femur. AJR Am J Roentgenol 1981;137:581-584. Watts HG: Fractures of the pelvis in children, Orthop Clin North Am 1976; 5-624, 2010 Pediatric Orthopaedic Examination Answer Book * 83 Question 97 Congenital anomalies of the vertebral column are associated frequently with other organ system problems. In addition to radiographs of the spine, what other screening tests should be ordered? Spinal MRI, coagulation panel Liver enzymes, coagulation pane! Renal ultrasound, upper and lower GI Cardiac evaluation/echocardiogram, upper and lower GI Renal ultrasound, cardiac evaluation/echocardiogram, spinal MRI yrere PREFERRED RESPONSE: 5 DISCUSSION: Approximately 60% of patients with congenital anomalies of the spine have other associated findings. The spine develops around the same time as the cardiovascular system, the genitourinary system, and the musculoskeletal system. Around 20% of patients with congenital scoliosis have an associated urologic abnormality. Approximately 25% of patients with congenital scoliosis have an associated cardiac defect. Spinal cord abnormalities in one study occurred in approximately 37% of patients with congenital scoliosis. REFERENCES: Basu PS, Elsebaie H, Noordeen MH: Congenital spinal deformity: A comprehensive assessment at presentation. Spine 2002;27:2255-2259. Ferguson RL: Medical and congenital comorbidities associated with spinal deformities in the immature spine. J Bone Joint Surg Am 2007;89:34-41 McMaster MJ, Ohtsuka K: The natural history of congenital scoliosis: A study of two hundred and fifty- one patients. J Bone Joint Surg Am 1982;64:1128-1147. 84+ American Academy of Orthopaedic Surgeons Figure 982 Figure 980 Question 98 A5-month-old child was referred for evaluation of right lower extremity bowing. The child’s developmental history is normal and there is no pain or history of trauma. Figures 98a and 98b show AP and lateral radiographs of the lower extremities. What is the natural progression of this condition? 1. Spontaneous resolution of the bowing by age 2 years without residual deformity 2. Spontaneous improvement or resolution of the bowing with a resulting limb-length inequality at maturity 3. No change in the condition over time 4. Spontaneous fracture in the area of the bo 5. Spread of the abnormal bone until the entire tibia is involved PREFERRED RESPONSE: 4 DISCUSSION: The patient has anterior lateral bowing of the tibia with intramedullary sclerosis at the site of the deformity. This is the prefracture stage of congenital pseudoarthrosis of the tibia and the child is at risk for spontaneous fracture and nonunion, A clamshell orthosis may prevent or delay fracture. Treatment of established nonunion ranges from bone grafting and intramedullary nailing to Ilizarov treatment, vascularized fibula bone grafting, or amputation. REFERENCES: Herring JA: Disorders of the leg, bowing of the tibia, in Herring JA (ed): Tachdjian’s Pediatric Orthopaedics, ed 4. Philadelphia, PA, WB Saunders, 2008, pp 1007-1023. Ohnishi I, Sato W, Matsuyama J, et al: Treatment of congenital pseudarthrosis of the tibia: A multicenter study in Japan. J Pediatr Orthop 2005;25:219-224, 2010 Pediatric Orthopaedic Examination Answer Book + 85 Question 99 ‘A 12-year-old girl is seen for left ankle pain. Radiographs reveal osteochondritis dissecans (OCD) involving the talus. What should the parents be told regarding management? 1, No treatment is required because spontaneous healing is common. 2. Nonsurgical management typically relieves pain and results in radiographic healing in less than 12 weeks, 3. Nonsurgical management frequently relieves pain but often may not result in radiographic healing even 6 months after treatment. 4. Hyperbaric oxygen treatment is helpful 5. Ankle fusion is frequently necessary. PREFERRED RESPONSE: 3 DISCUSSION: Nonsurgical management of OCD of the talus in skeletally immature individuals frequently results in a fairly rapid decrease in symptoms, but radiographic abnormalities can frequently be found even 6 months after treatment. Spontaneous resolution of this condition is rare. Hyperbaric oxygen treatment has not been shown to be beneficial for this condition. Progression of the condition to the point of requiring ankle fusion is rare. REFERENCES: Perumal V, Wall E, Babekir N: Juvenile osteochondritis dissecans of the talus. J Pediat Orthop 2007;27:821-825. Letts M, Davidson D, Ahmer A: Osteochondritis dissecans of the talus in children. J Pediatr Orthop 2003;23:617-625. Question 100 ‘A4-year-old child has droopy shoulders. Examination shows that the child has a large head, short stature, and a narrow chest. Radiographs of the spine and chest show absent clavicles, delayed ossification of the pubis and ischium, and mild coxa vara. What is the inheritance pattern for this condition? Autosomal dominant Autosomal recessive Sex-linked recessive Sex-linked dominant No inheritance pattem veep PREFERRED RESPONSE: | DISCUSSION: The child has the clinical and radiographic features of cleidocranial dysostosis. This is a disorder of bones formed by intramembranous ossification. It is inherited as an autosomal-dominant condition. About two thirds of cases are familial. REFERENCES: Dietz FR, Mathews KD: Update on the genetic bases of disorders with orthopacdic manifestations. J Joint Bone Surg Am 1996;78:1583-1598. Lee B, Thirunavukkarasu K, Zhou L, et al: Missense mutations abolishing DNA binding of osteoblast- specific transcription factor OSF2/CBFA 1 in cleidocranial dysplasia. Nat Genet 1997;16:307-310.

You might also like